*NURSING > EXAM > Chapter 10—Caring for Families (download to score an A) (All)

Chapter 10—Caring for Families (download to score an A)

Document Content and Description Below

Chapter 10—Caring for Families A nurse is assessing the family unit to determine the family’s ability to adapt to the change of a member having surgery. Which area is the nurse monitoring? ... A nurse reviews the current trends affecting the family. Which trend will the nurse find? A spouse brings the children in to visit their mother in the hospital. The nurse asks how the family is doing. The husband states, “None of her jobs are getting done, and I don’t do those jobs, so the house and the kids are falling apart.” How will the nurse interpret this finding? A nurse cares for the family’s as well as the patient’s needs using available resources. Which approach is the nurse using? A nurse is caring for a patient who needs constant care in the home setting and for whom most of the care is provided by the patient’s family. Which action should the nurse take to help relieve stress? A nurse is working with a patient. When the nurse asks about family members, the patient states that it includes my spouse, children, and aunt and uncle. How will the nurse describe this type of family? A nurse is assessing a child that lives in a car with family members who presents to the emergency department. Which area should the nurse assess closely? The nurse is interviewing a patient who is being admitted to the hospital. The patient’s family went home before the nurse’s interview. The nurse asks the patient, “Who decides when to come to the hospital?” What is the rationale for the nurse’s action? A nurse is caring for a patient from a motor vehicle accident. Which action by the unlicensed assistive personnel will cause the nurse to intervene? A nurse is using the family as context approach to provide care to a patient. What should the nurse do next? The nurse is caring for a patient in hospice. The nurse notes that the patient is getting adequate care, but the spouse is not sleeping well. The nurse also assesses the need for better family nutrition and meals assistance. The nurse discusses these needs with the patient and family and develops a plan of care with them using community resources. Which approach is the nurse using? The nurse is caring for an older adult patient who has no apparent family. When questioned about family and the definition of family, the patient states, “I have no family. They’re all gone.” When asked, “Who prepares your meals?” the patient states, “I do, or I go out.” Which approach should the nurse use for this patient? The nurse is caring for an older adult patient at home who requires teaching for dressing changes. The spouse and adult child are also involved in changing the dressing. Which statement by the nurse will most likely elicit a positive response from the patient and family? The nurse is providing discharge teaching for an older-adult patient who will need tube feedings at home. The spouse is the only source of care and states “I will not be able to perform the feedings due to arthritis.” Which action should the nurse take? A nurse is assessing threats concerning the family. Which areas will the nurse include in the assessment? (Select all that apply.) A nurse is assessing the realms of family life. Which processes will the nurse assess? (Select all that apply.) A nurse is focusing on the interactive processes of family life and is asking the patient questions. Match the questions the nurse will ask to the interactive process. Who is the “peacekeeper” of the family? Roles How are house rules established? Family nurturing How often does the family hug each other? Intimacy expression Who at your workplace is close to the family? Social suppor Chapter 16—Nursing Assessment The nurse is using critical thinking skills during the first phase of the nursing process. Which action indicates the nurse is in the first phase? Completes a comprehensive database A nurse is using the problem-oriented approach to data collection. Which action will the nurse take first? Focus on the patient’s presenting situation. After reviewing the database, the nurse discovers that the patient’s vital signs have not been recorded by the nursing assistive personnel (NAP). Which clinical decision should the nurse make? Ask the NAP to record the patient’s vital signs before administering medications. The nurse is gathering data on a patient. Which data will the nurse report as objective data? Respirations 16 A patient expresses fear of going home and being alone. Vital signs are stable and the incision is nearly completely healed. What can the nurse infer from the subjective data? The patient is apprehensive about discharge. Which method of data collection will the nurse use to establish a patient’s database? Performing a physical examination A nurse is gathering information about a patient’s habits and lifestyle patterns. Which method of data collection will the nurse use that will best obtain this information? Perform a thorough nursing health history. While interviewing an older female patient of Asian descent, the nurse notices that the patient looks at the ground when answering questions. What should the nurse do? Consider cultural differences during this assessment. A nurse has already set the agenda during a patient-centered interview. What will the nurse do next? Ask about the chief concerns or problems. The nurse is attempting to prompt the patient to elaborate on the reports of daytime fatigue. Which question should the nurse ask? “What reasons do you think are contributing to your fatigue?” A nurse is conducting a nursing health history. Which component will the nurse address? Patient expectations While the patient’s lower extremity, which is in a cast, is assessed, the patient tells the nurse about an inability to rest at night. The nurse disregards this information, thinking that no correlation has been noted between having a leg cast and developing restless sleep. Which action would have been best for the nurse to take? Ask the patient about usual sleep patterns and the onset of having difficulty resting. The nurse begins a shift assessment by examining a surgical dressing that is saturated with serosanguineous drainage on a patient who had open abdominal surgery yesterday (or 1 day ago). Which type of assessment approach is the nurse using? Problem-oriented assessment Which statement by a nurse indicates a good understanding about the differences between data validation and data interpretation? “Validation involves comparing data with other sources for accuracy.” Which scenario best illustrates the nurse using data validation when making a nursing clinical decision for a patient? The nurse determines to remove a wound dressing when the patient reveals the time of the last dressing change and notices old and new drainage. While completing an admission database, the nurse is interviewing a patient who states “I am allergic to latex.” Which action will the nurse take first? Ask the patient to describe the type of reaction. A patient verbalizes a low pain level of 2 out of 10 but exhibits extreme facial grimacing while moving around in bed. What is the nurse’s initial action in response to these observations? Ask the patient about the facial grimacing with movement. The nurse is interviewing a patient with a hearing deficit. Which area should the nurse use to conduct this interview? The patient’s room with the door closed A new nurse is completing an assessment on an 80-year-old patient who is alert and oriented. The patient’s daughter is present in the room. Which action by the nurse will require follow-up by the charge nurse? The nurse speaks only to the patient’s daughter. A nurse is completing an assessment. Which findings will the nurse report as subjective data? (Select all that apply.) Patient describing excitement about discharge Patient’s expression of fear regarding upcoming surgery A nurse is completing an assessment using the PQRST to obtain data about the patient’s chest pain. Match the questions to the components of the PQRST that the nurse will be using. Where is the pain located? Radiate What causes the pain? Provokes Does it come and go? Time What does the pain feel like? Quality What is the rating on a scale of 0 to 10? Severity Chapter 18—Planning Nursing Care The nurse completes a thorough assessment of a patient and analyzes the data to identify nursing diagnoses. Which step will the nurse take next in the nursing process? Planning A patient’s plan of care includes the goal of increasing mobility this shift. As the patient is ambulating to the bathroom at the beginning of the shift, the patient suffers a fall. Which initial action will the nurse take next to revise the plan of care? Assess the patient. Which information indicates a nurse has a good understanding of a goal? It is a broad statement describing a desired change in a patient’s behavior. A nurse is developing a care plan for a patient with a pelvic fracture on bed rest. Which goal statement is realistic for the nurse to assign to this patient? Patient will increase activity level this shift. The following statements are on a patient’s nursing care plan. Which statement will the nurse use as an outcome for a goal of care? The patient will verbalize a decreased pain level less than 3 on a 0 to 10 scale by the end of this shift. A charge nurse is reviewing outcome statements using the SMART approach. Which patient outcome statement will the charge nurse praise to the new nurse? The patient will feed self at all mealtimes today without reports of shortness of breath. A nursing assessment for a patient with a spinal cord injury leads to several pertinent nursing diagnoses. Which nursing diagnosis is the highest priority for this patient? Reflex urinary incontinence The new nurse is caring for six patients in this shift. After completing their assessments, the nurse asks where to begin in developing care plans for these patients. Which statement is an appropriate suggestion by another nurse? “Begin with the highest priority diagnoses, then select appropriate interventions.” A patient’s son decides to stay at the bedside while his father is confused. When developing the plan of care for this patient, what should the nurse do? Involve the son in the plan of care as much as possible. A nurse is caring for a patient with a nursing diagnosis of Constipation related to slowed gastrointestinal motility secondary to pain medications. Which outcome is most appropriate for the nurse to include in the plan of care? Patient will have one soft, formed bowel movement by end of shift. The nurse performs an intervention for a collaborative problem. Which type of intervention did the nurse perform? Interdependent A registered nurse administers pain medication to a patient suffering from fractured ribs. Which type of nursing intervention is this nurse implementing? Dependent Which action indicates the nurse is using a PICOT question to improve care for a patient? Implements interventions based on scientific research A nurse is developing a care plan. Which intervention is most appropriate for the nursing diagnostic statement Risk for loneliness related to impaired verbal communication? Provide the patient with a writing board each shift. A nurse is completing a care plan. Which intervention is most appropriate for the nursing diagnostic statement Impaired skin integrity related to shearing forces? Turn the patient every 2 hours, even hours. A patient has reduced muscle strength following a left-sided stroke and is at risk for falling. Which intervention is most appropriate for the nursing diagnostic statement Risk for falls? Assist patient into and out of bed every 4 hours or as tolerated. Which action will the nurse take after the plan of care for a patient is developed? Communicate the plan to all health care professionals involved in the patient’s care. A nurse is preparing to make a consult. In which order, beginning with the first step, will the nurse take? Identify the problem. Identify > Contact > Provide > Avoid > Discuss // ICPAD Contact the right professional, with the appropriate knowledge and expertise. Provide the consultant with relevant information about the problem. Avoid bias by not providing a lot of information based on opinion to the consultant. Discuss the findings and recommendation. A hospital’s wound nurse consultant made a recommendation for nurses on the unit about how to care for the patient’s dressing changes. Which action should the nurses take next? Include dressing change instructions and frequency in the care plan. A nurse is planning care for a patient with a nursing diagnosis of Impaired skin integrity. The patient needs many nursing interventions, including a dressing change, several intravenous antibiotics, and a walk. Which factors does the nurse consider when prioritizing interventions? (Select all that apply.) Rank all the patient’s nursing diagnoses in order of priority. Consider time as an influencing factor. Utilize critical thinking. A nurse is teaching the staff about the benefits of Nursing Outcomes Classification. Which information should the nurse include in the teaching session? (Select all that apply.) Adds objectivity to judging a patient’s progress Measures nursing care on a national and international level Chapter 19—Implementing Nursing Care A nurse is providing nursing care to patients after completing a care plan from nursing diagnoses. In which step of the nursing process is the nurse? Implementation The nurse is teaching a new nurse about protocols. Which information from the new nurse indicates a correct understanding of the teaching? Protocols assist the clinician in making decisions and choosing interventions for specific health care problems or conditions. The standing orders for a patient include acetaminophen 650 mg every 4 hours prn for headache. After assessing the patient, the nurse identifies the need for headache relief and determines that the patient has not had acetaminophen in the past 4 hours. Which action will the nurse take next? Administer the acetaminophen. Which action indicates a nurse is using critical thinking for implementation of nursing care to patients? Determines whether an intervention is correct and appropriate for the given situation A nurse is reviewing a patient’s care plan. Which information will the nurse identify as a nursing intervention? Provide assistance while the patient walks in the hallway twice this shift with crutches. A patient recovering from a leg fracture after a fall reports having dull pain in the affected leg and rates it as a 7 on a 0 to 10 scale. The patient is not able to walk around in the room with crutches because of leg discomfort. Which nursing intervention is priority? Administer pain medication. The nurse is caring for a patient who requires a complex dressing change. While in the patient’s room, the nurse decides to change the dressing. Which action will the nurse take just before changing the dressing? Assesses the patient’s readiness for the procedure A patient visiting with family members in the waiting area tells the nurse “I don’t feel good, especially in the stomach.” What should the nurse do? Ask the patient to return to the room, so the nurse can inspect the abdomen. A newly admitted patient who is morbidly obese asks the nurse for assistance to the bathroom for the first time. Which action should the nurse take initially? Review the patient’s activity orders. A new nurse is working in a unit that uses interdisciplinary collaboration. Which action will the nurse take? Develop good communication skills. Which action should the nurse take first during the initial phase of implementation? Reassess the patient. Vital signs for a patient reveal a high blood pressure of 187/100. Orders state to notify the health care provider for diastolic blood pressure greater than 90. What is the nurse’s first action? Assess the patient for other symptoms or problems, and then notify the health care provider. Which initial intervention is most appropriate for a patient who has a new onset of chest pain? Reassess the patient. A nurse is making initial rounds on patients. Which intervention for a patient with poor wound healing should the nurse perform first? Observe wound appearance and edges. The nurse establishes trust and talks with a school-aged patient before administering an injection. Which type of implementation skill is the nurse using? Interpersonal The nurse inserts an intravenous (IV) catheter using the correct technique and following the recommended steps according to standards of care and hospital policy. Which type of implementation skill is the nurse using? Psychomotor A staff development nurse is providing an inservice for other nurses to educate them about the Nursing Interventions Classification (NIC) system. During the inservice, which statement made by one of the nurses in the room requires the staff development nurse to clarify the information provided? “This system can help medical students determine the cost of the care they provide to patients.” The nurse is intervening for a family member with role strain. Which direct care nursing intervention is most appropriate? Counseling about respite care options The nurse is intervening for a patient that has a risk for a urinary infection. Which direct care nursing intervention is most appropriate? Teaches proper handwashing technique The nurse is revising the care plan. In which order will the nurse perform the tasks, beginning with the first step? Revise > Delete > Revise > Choose //// RDVC Revise the assessment column. Delete irrelevant nursing diagnoses. Revise specific interventions. Choose the evaluation method. A nurse is implementing interventions for a group of patients. Which actions are nursing interventions? (Select all that apply.) Reposition a patient who is on bed rest. Teach a patient preoperative exercises. Transfer a patient to another hospital unit. A nurse is providing nursing care to a group of patients. Which actions are direct care interventions? (Select all that apply.) Ambulating a patient Inserting a feeding tube Performing resuscitation Teaching about medications A nurse is preparing to carry out interventions. Which resources will the nurse make sure are available? (Select all that apply.) Equipment Safe environment Assistive personnel Which interventions are appropriate for a patient with diabetes and poor wound healing? (Select all that apply.) Perform dressing changes twice a day as ordered. Teach the patient about signs and symptoms of infection. Instruct the family about how to perform dressing changes. Administer medications to control the patient’s blood sugar as ordered. Chapter 20—Evaluation A nurse determines that the patient’s condition has improved and has met expected outcomes. Which step of the nursing process is the nurse exhibiting? Evaluation A nurse completes a thorough database and carries out nursing interventions based on priority diagnoses. Which action will the nurse take next? Evaluation A new nurse asks the preceptor to describe the primary purpose of evaluation. Which statement made by the nursing preceptor is most accurate? “Nurses use evaluation to determine the effectiveness of nursing care.” After assessing the patient and identifying the need for headache relief, the nurse administers acetaminophen for the patient’s headache. Which action by the nurse is priority for this patient? Reassess the patient’s pain level in 30 minutes. A nurse is getting ready to discharge a patient who has a problem with physical mobility. What does the nurse need to do before discontinuing the patient’s plan of care? Evaluate whether patient goals and outcomes have been met. The nurse is evaluating whether patient goals and outcomes have been met for a patient with physical mobility problems due to a fractured leg. Which finding indicates the patient has met an expected outcome? The patient is able to ambulate in the hallway with crutches. The nurse is evaluating whether a patient’s turning schedule was effective in preventing the formation of pressure ulcers. Which finding indicates success of the turning schedule? Absence of skin breakdown A nurse has instituted a turn schedule for a patient to prevent skin breakdown. Upon evaluation, the nurse finds that the patient has a stage II pressure ulcer on the buttocks. Which action will the nurse take next? Reassess the patient and situation. A new nurse is confused about using evaluative measures when caring for patients and asks the charge nurse for an explanation. Which response by the charge nurse is most accurate? “Evaluative measures include assessment data used to determine whether patients have met their expected outcomes and goals.” The nurse is caring for a patient who has an open wound and is evaluating the progress of wound healing. Which priority action will the nurse take? Measure the wound and observe for redness, swelling, or drainage. The nurse is caring for a patient who has an order to change a dressing twice a day, at 0600 and 1800. At 1400, the nurse notices that the dressing is saturated and leaking. What is the nurse’s next action? Revise the plan of care and change the dressing now. A goal for a patient with diabetes is to demonstrate effective coping skills. Which patient behavior will indicate to the nurse achievement of this outcome? States feels better after talking with family and friends A nurse is providing education to a patient about self-administering subcutaneous injections. The patient demonstrates the self- injection. Which type of indicator did the nurse evaluate? Health behavior A nurse is evaluating the goal of acceptance of body image in a young teenage girl. Which statement made by the patient is the best indicator of progress toward the goal? “I’ll wear the blue dress. It matches my eyes.” A nurse is evaluating goals and expected outcomes for a confused patient. Which finding indicates positive progress toward resolving the confusion? Patient correctly states names of family members in the room. A nurse identifies a fall risk when assessing a patient upon admission. The nurse and the patient agree that the goal is for the patient to remain free from falls. However, the patient fell just before shift change. Which action is the nurse’s priority when evaluating the patient? Identify factors interfering with goal achievement. A patient was recently diagnosed with pneumonia. The nurse and the patient have established a goal that the patient will not experience shortness of breath with activity in 3 days with an expected outcome of having no secretions present in the lungs in 48 hours. Which evaluative measure will the nurse use to demonstrate progress toward this goal? Lungs clear to auscultation following use of inhaler A nurse is evaluating an expected outcome for a patient that states heart rate will be less than 80 beats/min by 12/3. Which finding will alert the nurse that the goal has been met? Heart rate 78 beats/min on 12/3 A nurse is modifying a patient’s care plan after evaluation of patient care. In which order, starting with the first step, will the nurse perform the tasks? Reassess > Revise > Change > Administer > Retake // ReaRevCARet Reassess blood pressure reading. Revise nursing diagnosis. Change goal to blood pressure less than 140/90. Administer new blood pressure medication. Retake blood pressure after medication. A nurse is caring for a group of patients. Which evaluative measures will the nurse use to determine a patient’s responses to nursing care? (Select all that apply.) Observations of wound healing Daily blood pressure measurements Findings of respiratory rate and depth Patient’s subjective report of feelings about a new diagnosis of cancer Which nursing actions will the nurse perform in the evaluation phase of the nursing process? (Select all that apply.) Determine whether outcomes or standards are met. Document results of goal achievement. Use self-reflection and correct errors. Chapter 24—Communication Which types of nurses make the best communicators with patients? Those who develop critical thinking skills A nurse believes that the nurse-patient relationship is a partnership and that both are equal participants. Which term should the nurse use to describe this belief? Mutuality A nurse wants to present information about flu immunizations to the older adults in the community. Which type of communication should the nurse use? Public A nurse is using therapeutic communication with a patient. Which technique will the nurse use to ensure effective communication? Electronic communication to assess a patient in another city A nurse is standing beside the patient’s bed. Nurse: How are you doing? ... Patient: I don’t feel good. Which element will the nurse identify as feedback? I don’t feel good. A nurse is sitting at the patient’s bedside taking a nursing history. Which zone of personal space is the nurse using? Personal 18in – 4 ft A smiling patient angrily states, “I will not cough and deep breathe.” How will the nurse interpret this finding? The patient’s affect is inappropriate. The nurse asks a patient where the pain is, and the patient responds by pointing to the area of pain. Which form of communication did the patient use? Nonverbal A patient has been admitted to the hospital numerous times. The nurse asks the patient to share a personal story about the care that has been received. Which interaction is the nurse using? Narrative Before meeting the patient, a nurse talks to other caregivers about the patient. Which phase of the helping relationship is the nurse in with this patient? Preinteraction During the initial home visit, a home health nurse lets the patient know that the visits are expected to end in about a month. Which phase of the helping relationship is the nurse in with this patient? Orientation A nurse and a patient work on strategies to reduce weight. Which phase of the helping relationship is the nurse in with this patient? Working A nurse uses SBAR when providing a hands-off report to the oncoming shift. What is the rationale for the nurse’s action? To standardize communication A patient was admitted 2 days ago with pneumonia and a history of angina. The patient is now having chest pain with a pulse rate of 108. Which piece of data will the nurse use for “B” when using SBAR? History of angina A patient just received a diagnosis of cancer. Which statement by the nurse demonstrates empathy? “This must be hard news to hear.” A nurse is taking a history on a patient who cannot speak English. Which action will the nurse take? Obtain an interpreter. A nurse is using SOLER to facilitate active listening. Which technique should the nurse use for R? Relax An older-adult patient is wearing a hearing aid. Which technique should the nurse use to facilitate communication? Turn off the television. When making rounds, the nurse finds a patient who is not able to sleep because of surgery in the morning. Which therapeutic response is most appropriate? “It must be difficult not to know what the surgeon will find. What can I do to help?” Which situation will cause the nurse to intervene and follow up on the nursing assistive personnel’s (NAP) behavior? The nursing assistive personnel is calling the older-adult patient “honey.” A confused older-adult patient is wearing thick glasses and a hearing aid. Which intervention is the priority to facilitate communication? Allow time for the patient to respond. The staff is having a hard time getting an older-adult patient to communicate. Which technique should the nurse suggest the staff use? Allow the patient to reminisce. A nurse is implementing nursing care measures for patients’ special communication needs. Which patient will need the most nursing care measures? The patient who is dyspneic, anxious, and has a tracheostomy A patient is aphasic, and the nurse notices that the patient’s hands shake intermittently. Which nursing action is most appropriate to facilitate communication? Use a picture board. Which behavior indicates the nurse is using a process recording correctly to enhance communication with patients? Self-examines personal communication skills A patient says, “You are the worst nurse I have ever had.” Which response by the nurse is most assertive? “I feel uncomfortable hearing that statement.” Which behaviors indicate the nurse is using critical thinking standards when communicating with patients? (Select all that apply.) Uses humility Portrays self-confidence Demonstrates independent attitude A nurse is implementing nursing care measures for patients with challenging communication issues. Which types of patients will need these nursing care measures? (Select all that apply.) A child who is developmentally delayed An older-adult patient who is demanding A female patient who is outgoing and flirty A teenager frightened by the prospect of impending surgery A nurse is using AIDET to communicate with patients and families. Match the letters of the acronym to the behavior a nurse will use. Nurse describes procedures and tests. E Nurse lets the patient know how long the procedure will last. D Nurse recognizes the person with a positive attitude. A Nurse thanks the patient. T Nurse tells the patient “I am an RN and will be managing your care.” I Chapter 27—Patient Safety and Quality A home health nurse is performing a home assessment for safety. Which comment by the patient will cause the nurse to follow up? “When it is cold outside in the winter, I will use a nonvented furnace.” The nurse is caring for an older-adult patient admitted with nausea, vomiting, and diarrhea due to food poisoning. The nurse completes the health history. Which priority concern will require collaboration with social services to address the patient’s health care needs? The electricity was turned off 3 days ago. The patient has been diagnosed with a respiratory illness and reports shortness of breath. The nurse adjusts the temperature to facilitate the comfort of the patient. At which temperature range will the nurse set the thermostat? 65° to 75° F A homeless adult patient presents to the emergency department. The nurse obtains the following vital signs: temperature 94.8° F, blood pressure 106/56, apical pulse 58, and respiratory rate 12. Which vital sign should the nurse address immediately? Temperature A nurse is teaching the patient and family about wound care. Which technique will the nurse teach to best prevent transmission of pathogens? Wash hands The nurse is monitoring for Never Events. Which finding indicates the nurse will report a Never Event? A surgical sponge is left in the patient’s incision. The nurse discovers a patient on the floor. The patient states that he fell out of bed. The nurse assesses the patient and places the patient back in bed. Which action should the nurse take next? Notify the health care provider. When making rounds the nurse observes a purple wristband on a patient’s wrist. How will the nurse interpret this finding? The patient has do not resuscitate preferences. A nurse reviews the history of a newly admitted patient. Which finding will alert the nurse that the patient is at risk for falls? Orthostatic hypotension The nurse is assessing a patient for lead poisoning. Which patient is the nurse most likely assessing? Toddler A nurse is teaching a community group of school-aged parents about safety. Which safety item is most important for the nurse to include in the teaching session? Proper fit of a bicycle helmet The nurse is presenting an educational session on safety for parents of adolescents. Which information will the nurse include in the teaching session? Increased aggressiveness and blood spots on clothing may indicate substance abuse. The nurse is discussing about threats to adult safety with a college group. Which statement by a group member indicates understanding of the topic? “Smoking even at parties is not good for my body.” The nurse is teaching a group of older adults at an assisted-living facility about age-related physiological changes affecting safety. Which question would be most important for the nurse to ask this group? “Are you able to hear the tornado sirens in your area?” The nurse is caring for a hospitalized patient. Which behavior alerts the nurse to consider the need for a restraint? The patient continues to remove the nasogastric tube. The nurse is trying to use alternatives rather than restrain a patient. Which finding will cause the nurse to determine the alternative is working? The patient folds three washcloths over and over. The nurse is caring for a patient who suddenly becomes confused and tries to remove an intravenous (IV) infusion. Which priority action will the nurse take? Assess the patient. The nurse is monitoring for the four categories of risk that have been identified in the health care environment. Which examples will alert the nurse that these safety risks are occurring? Wet floors unmarked, patient pinching fingers in door, failure to use lift for patient, and alarms not functioning properly Which activity will cause the nurse to monitor for equipment-related accidents? Uses a patient-controlled analgesic pump A patient is admitted and is placed on fall precautions. The nurse teaches the patient and family about fall precautions. Which action will the nurse take? Keep the patient on fall risk until discharge. A nurse is inserting a urinary catheter. Which technique will the nurse use to prevent a procedure-related accident? Surgical asepsis A nurse is providing care to a patient. Which action indicates the nurse is following the National Patient Safety Goals? Uses medication bar coding when administering medications During the admission assessment, the nurse assesses the patient for fall risk. Which finding will alert the nurse to an increased risk for falls? The patient takes a hypnotic. A patient may need restraints. Which task can the nurse delegate to a nursing assistive personnel? Applying the restraint A patient has an ankle restraint applied. Upon assessment the nurse finds the toes a light blue color. Which action will the nurse take next? Remove the restraint. The emergency department has been notified of a potential bioterrorism attack. Which action by the nurse is priority? Manage all patients using standard precautions. The patient is confused, is trying to get out of bed, and is pulling at the intravenous infusion tubing. Which nursing diagnosis will the nurse add to the care plan? Risk for injury A confused patient is restless and continues to try to remove the oxygen cannula and urinary catheter. What is the priority nursing diagnosis and intervention to implement for this patient? Risk for injury: Check on patient every 15 minutes. The patient applies sequential compression devices after going to the bathroom. The nurse checks the patient’s application of the devices and finds that they have been put on upside down. Which nursing diagnosis will the nurse add to the patient’s plan of care? Deficient knowledge The nurse enters the patient’s room and notices a small fire in the headlight above the patient’s bed. In which order will the nurse perform the steps, beginning with the first one? Remove the patient. // Remove > Pull > Close > Use // RPCU Pull the alarm. Close doors and windows. Use the fire extinguisher. The nurse is providing information regarding safety and accidental poisoning to a grandparent who will be taking custody of a 1-year- old grandchild. Which comment by the grandparent will cause the nurse to intervene? “If my grandchild eats a plant, I should provide syrup of ipecac.” A home health nurse is assessing a family’s home after the birth of an infant. A toddler also lives in the home. Which finding will cause the nurse to follow up? Plastic grocery bags are neatly stored under the counter. Which patient will the nurse see first? A 56-year-old patient with oxygen using an electric razor for grooming A home health nurse is teaching a family to prevent electrical shock. Which information will the nurse include in the teaching session? Disconnect items before cleaning. The nurse has placed a yellow armband on a 70-year-old patient. Which observation by the nurse will indicate the patient has an understanding of this action? The patient wears the red nonslip footwear. An older-adult patient is using a wheelchair to attend a physical therapy session. Which action by the nurse indicates safe transport of the patient? Backs wheelchair into elevator, leading with large rear wheels first A home health nurse is assessing the home for fire safety. Which information from the family will cause the nurse to intervene? (Select all that apply.) Smoking in bed helps me relax and fall asleep. We use the same space heater my grandparents used. We use the RACE method when using the fire extinguisher. The nurse is caring for an older adult who presents to the clinic after a fall. The nurse reviews fall prevention in the home. Which information will the nurse include in the teaching session? (Select all that apply.) Walk to the mailbox in the summer. Encourage yearly eye examinations. Keep pathways clutter free. A patient requires restraints after alternatives are not successful. The nurse is reviewing the orders. Which findings indicate to the nurse the order is legal and appropriate for safe care? (Select all that apply.) Health care provider writes the type and location of the restraint. Health care provider performs a face-to-face assessment prior to the order. Health care provider specifies the duration and circumstances under which the restraint will be used. The nurse is performing the “Timed Get Up and Go (TUG)” assessment. Which actions will the nurse take? (Select all that apply.) Instructs the patient to walk 10 feet as quickly and safely as possible Observes for unsteadiness in patient’s gait Allows the patient a practice trial The nurse is completing an admission history on a new home health patient. The patient has been experiencing seizures as the result of a recent brain injury. Which interventions should the nurse utilize for this patient and family? (Select all that apply.) Discuss with the family steps to take if the seizure does not discontinue. Instruct the family to reorient and reassure the patient after consciousness is regained. The nurse is assessing a patient who reports a previous fall and is using the SPLATT acronym. Which questions will the nurse ask the patient? (Select all that apply.) Where did you fall? What time did the fall occur? What were you doing when you fell? What types of injuries occurred after the fall? The nurse is caring for a group of medical-surgical patients. The unit has been notified of a fire on an adjacent wing of the hospital. The nurse quickly formulates a plan to keep the patients safe. Which actions will the nurse take? (Select all that apply.) Close all doors. Note evacuation routes. Note oxygen shut-offs. Move bedridden patients in their bed. The nurse is caring for a patient in restraints. Which essential information will the nurse document in the patient’s medical record to provide safe care? (Select all that apply.) Patient is placed in bilateral wrist restraints at 0815. Bilateral radial pulses present, 2+, hands warm to touch Attempts to distract the patient with television are unsuccessful. Released from restraints, active range-of-motion exercises completed Chapter 28 — Immobility A nurse is assessing body alignment. What is the nurse monitoring? The relationship of one body part to another while in different positions A nurse is providing range of motion to the shoulder and must perform external rotation. Which action will the nurse take? Moves patient’s arm until thumb is upward and lateral to head with elbow flexed A nurse is providing passive range of motion (ROM) for a patient with impaired mobility. Which technique will the nurse use for each movement? Each movement is moved just to the point of resistance by the nurse. A nurse is performing passive range of motion (ROM) and splinting on an at-risk patient. Which finding will indicate goal achievement for the nurse’s action? Prevention of joint contractures A nurse is preparing to reposition a patient. Which task can the nurse delegate to the nursing assistive personnel? Changing the patient’s position A nurse is preparing to assess a patient for orthostatic hypotension. Which piece of equipment will the nurse obtain to assess for this condition? Blood pressure cuff The patient has been in bed for several days and needs to be ambulated. Which action will the nurse take first? Dangle the patient at the bedside. A nurse reviews an immobilized patient’s laboratory results and discovers hypercalcemia. Which condition will the nurse monitor for most closely in this patient? Renal calculi A nurse is caring for an immobile patient. Which metabolic alteration will the nurse monitor for in this patient? Altered nutrient metabolism A nurse is preparing a care plan for a patient who is immobile. Which psychosocial aspect will the nurse consider? Loss of hope The nurse is preparing to lift a patient. Which action will the nurse take first? Assess weight and determine assistance needs. The nurse is caring for an older-adult patient who has been diagnosed with a stroke. Which intervention will the nurse add to the care plan? Encourage the patient to perform as many self-care activities as possible. The nurse is observing the way a patient walks. Which aspect is the nurse assessing? Gait A nurse is assessing the body alignment of a standing patient. Which finding will the nurse report as normal? When observed laterally, the spinal curves align in a reversed “S” pattern. The nurse is evaluating the body alignment of a patient in the sitting position. Which observation bythe nurse will indicate a normal finding? Both feet are supported on the floor with ankles flexed. The nurse is assessing body alignment for a patient who is immobilized. Which patient position will the nurse use? Lateral position The nurse is assessing the patient for respiratory complications of immobility. Which action will the nurse take when assessing the respiratory system? Auscultate the entire lung region to assess lung sounds. The nurse is assessing an immobile patient for deep vein thromboses (DVTs). Which action will the nurse take? Measure the calf circumference of both legs. A nurse is assessing the skin of an immobilized patient. What will the nurse do? Use a standardized tool such as the Braden Scale. The nurse is caring for an older-adult patient with a diagnosis of urinary tract infection (UTI). Upon assessment the nurse finds the patient confused and agitated. How will the nurse interpret these assessment findings? These are common manifestation with UTIs. A patient has damage to the cerebellum. Which disorder is most important for the nurse to assess? Imbalance Which patient will cause the nurse to select a nursing diagnosis of Impaired physical mobility for a care plan? A patient who is not completely immobile The patient has the nursing diagnosis of Impaired physical mobility related to pain in the left shoulder. Which priority action will the nurse take? Assist the patient with comfort measures. A nurse is developing an individualized plan of care for a patient. Which action is important for the nurse to take? Establish goals that are measurable and realistic. Which behavior indicates the nurse is using a team approach when caring for a patient who is experiencing alterations in mobility? Consults physical therapy for strengthening exercises in the extremities The patient is being admitted to the neurological unit with a diagnosis of stroke. When will the nurse begin discharge planning? At the time of admission Which goal is most appropriate for a patient who has had a total hip replacement? The patient will walk 100 feet using a walker by the time of discharge. The nurse is working on an orthopedic rehabilitation unit that requires lifting and positioning of patients. Which personal injury will the nurse most likely try to prevent? Back A nurse is caring for a patient with osteoporosis and lactose intolerance. What will the nurse do? Monitor intake of vitamin D. A nurse is providing care to a group of patients. Which patient will the nurse see first? A patient with a hip replacement on prolonged bed rest reporting chest pain and dyspnea The patient is immobilized after undergoing hip replacement surgery. Which finding will alert the nurse to monitor for hemorrhage in this patient? Low-molecular-weight heparin doses The nurse needs to move a patient up in bed using a drawsheet. The nurse has another nurse helping. In which order will the nurses perform the steps, beginning with the first one? Position one nurse at each side of the bed. // Position > Place sheet > Grasp > Place > Flex > Move Place the drawsheet under the patient from shoulder to thigh. Grasp the drawsheet firmly near the patient. Place your feet apart with a forward-backward stance. Flex knees and hips and on count of three shift weight from the front to back leg. Move the patient and drawsheet to the desired position. The nurse is caring for a patient who needs to be placed in the prone position. Which action will the nurse take? Place pillow under the patient’s abdomen after turning. The nurse is caring for a patient with a spinal cord injury and notices that the patient’s hips have a tendency to rotate externally when the patient is supine. Which device will the nurse use to help prevent injury secondary to this rotation? A trochanter roll The patient is unable to move self and needs to be pulled up in bed. What will the nurse do to make this procedure safe? Use a friction-reducing device. The nurse is caring for a patient who is immobile and needs to be turned every 2 hours. The patient has poor lower extremity circulation, and the nurse is concerned about irritation of the patient’s toes. Which device will the nurse use? A foot cradle A nurse delegates a position change to a nursing assistive personnel. The nurse instructs the NAP to place the patient in the lateral position. Which finding by the nurse indicates a correct outcome? Patient is lying on side. A nurse is evaluating care of an immobilized patient. Which action will the nurse take? Compare the patient’s actual outcomes with the outcomes in the care plan. A nurse is supervising the logrolling of a patient. To which patient is the nurse most likely providing care? A patient with neck surgery The nurse is providing teaching to an immobilized patient with impaired skin integrity about diet. Which diet will the nurse recommend? High protein, high calorie The nurse is caring for a patient who has had a stroke causing total paralysis of the right side. To help maintain joint function and minimize the disability from contractures, passive ROM will be initiated. When should the nurse begin this therapy? As soon as the ability to move is lost The nurse is admitting a patient who has been diagnosed as having had a stroke. The health care provider writes orders for “ROM as needed.” What should the nurse do next? Further assess the patient. A nurse is assessing pressure points in a patient placed in the Sims’ position. Which areas will the nurse observe? Ileum, clavicle, knees The patient is admitted to a skilled care unit for rehabilitation after the surgical procedure of fixation of a fractured left hip. The patient’s nursing diagnosis is Impaired physical mobility related to musculoskeletal impairment from surgery and pain with movement. The patient is able to use a walker but needs assistance ambulating and transferring from the bed to the chair. Which nursing intervention is most appropriate for this patient? Assist with ambulation and measure how far the patient walks. The patient has been diagnosed with a spinal cord injury and needs to be repositioned using the logrolling technique. Which technique will the nurse use for logrolling? Use at least three people. Upon assessment a nurse discovers that a patient has erythema. Which actions will the nurse take? (Select all that apply.) Consult a dietitian. Increase frequency of turning. Place on pressure-relieving mattress. The nurse is caring for a patient with impaired physical mobility. Which potential complications will the nurse monitor for in this patient? (Select all that apply.) Footdrop Hypostatic pneumonia Impaired skin integrity The nurse is caring for a patient who has had a recent stroke and is paralyzed on the left side. The patient has no respiratory or cardiac issues but cannot walk. The patient cannot button a shirt and cannot feed self due to being left-handed and becomes frustrated very easily. The patient has been eating very little and has lost 2 lbs. The patient asks the nurse, “How can I go home like this? I’m not getting better.” Which health care team members will the nurse need to consult? (Select all that apply.) Dietitian Physical therapist Occupational therapist Upon assessment a nurse discovers postural abnormalities on several patients. Match the abnormalities to the findings the nurse observed. Lordosis - Exaggeration of anterior convex curve of lumbar spine Kyphosis - Increased convexity in curvature of thoracic spine Scoliosis - Lateral-S- or C-shaped spinal column with vertebral rotation Genu valgum - Legs curved inward so knees come together as person walks Genu varum - One or both legs bent outward at knee Torticollis - Inclining of head to affected side Chapter 29 — Infection Prevention and Control The nurse and a new nurse in orientation are caring for a patient with pneumonia. Which statement by the new nurse will indicate a correct understanding of this condition? “An infectious disease like pneumonia may not pose a risk to others.” The patient and the nurse are discussing Rickettsia rickettsii—Rocky Mountain spotted fever. Which patient statement to the nurse indicates understanding regarding the mode of transmission for this disease? “When camping, I will wear insect repellent.” The nurse is providing an educational session for a group of preschool workers. The nurse reminds the group about the most important thing to do to prevent the spread of infection. Which information did the nurse share with the preschool workers? Wash their hands between each interaction with children. The nurse is admitting a patient with an infectious disease process. Which question will be most appropriate for a nurse to ask about the patient’s susceptibility to this infectious process? “Do you have a chronic disease?” The patient experienced a surgical procedure, and Betadine was utilized as the surgical prep. Two days postoperatively, the nurse’s assessment indicates that the incision is red and has a small amount of purulent drainage. The patient reports tenderness at the incision site. The patient’s temperature is 100.5° F, and the WBC is 10,500/mm3. Which action should the nurse take first? Utilize SBAR to notify the primary health care provider The nurse is providing an education session to an adult community group about the effects of smoking on infection. Which information is most important for the nurse to include in the educational session? Smoking affects the cilia lining the upper airways in the lungs. A female adult patient presents to the clinic with reports of a white discharge and itching in the vaginal area. A nurse is taking a health history. Which question is the priority? “What medications are you currently taking?” The nurse is caring for a school-aged child who has injured the right leg after a bicycle accident. Which signs and symptoms will the nurse assess for to determine if the child is experiencing a localized inflammatory response? Edema, redness, tenderness, and loss of function Which interventions utilized by the nurse will indicate the ability to recognize a localized inflammatory response? Rest, ice, and elevation The nurse is caring for a group of medical-surgical patients. Which patient is most at risk for developing an infection? A patient who is recovering from a right total hip surgery The nurse is caring for a patient with leukemia and is preparing to provide fluids through a vascular access (IV) device. Which nursing intervention is a priority in this procedure? Maintain surgical aseptic technique. The nurse is caring for an adult patient in the clinic who has been evacuated and is a victim of flooding. The nurse teaches the patient about rest, exercise, and eating properly and how to utilize deep breathing and visualization. What is the primary rationale for the nurse’s actions related to the teaching? Stress for long periods of time can lead to exhaustion and decreased resistance to infection. The nurse is caring for a patient who is susceptible to infection. Which instruction will the nurse include in an educational session to decrease the risk of infection? Teaching the patient to select nutritious foods A diabetic patient presents to the clinic for a dressing change. The wound is located on the right foot and has purulent yellow drainage. Which action will the nurse take to prevent the spread of infection? Don gloves and other appropriate personal protective equipment A patient presents with pneumonia. Which priority intervention should be included in the plan of care for this patient? Observe the patient for decreased activity tolerance The nurse is caring for a patient in an intensive care unit who needs a bath. Which priorityaction will the nurse take to decrease the potential for a health care–associated infection? Use a chlorhexidine wash The infection control nurse is reviewing data for the medical-surgical unit. The nurse notices an increase in postoperative infections from Aspergillus. Which type of health care–associated infection will the nurse report? Exogenous The patient has contracted a urinary tract infection (UTI) while in the hospital. Which action will most likely increase the risk of a patient contracting a UTI? Allowing the drainage bag port to touch the graduated receptacle Which nursing action will most likely increase a patient’s risk for developing a health care–associated infection? Uses a clean technique for inserting a urinary catheter The nurse is caring for a patient in labor and delivery. When near completing an assessment of the patient’s cervix, the electronic infusion device being used on the intravenous (IV) infusion alarms. Which sequence of actions is most appropriate for the nurse to take? Complete the assessment, remove gloves, wash hands, and assess the intravenous infusion. The nurse is dressed and is preparing to care for a patient in the perioperative area. The nurse has scrubbed hands and has donned a sterile gown and gloves. Which action will indicate a break in sterile technique? Touching clean protective eyewear The nurse is caring for a patient with an incision. Which actions will best indicate an understanding of medical and surgical asepsis for a sterile dressing change? Utilizing clean gloves to remove the dressing and sterile supplies for the new dressing The nurse is caring for a patient in the endoscopy area. The nurse observes the technician performing these tasks. Which observation will require the nurse to intervene? Removing gloves to transfer the endoscope The nurse is caring for a patient who is at risk for infection. Which action by the nurse indicates correct understanding about standard precautions? Wears eyewear when emptying the urinary drainage bag The nurse is caring for a patient who has just delivered a neonate. The nurse is checking the patient for excessive vaginal drainage. Which precaution will the nurse use? Standard The nurse is caring for a patient in the hospital. The nurse observes the nursing assistive personnel (NAP) turning off the handle faucet with bare hands. Which professional practice principle supports the need for follow-up with the NAP? The nurse is responsible for providing a safe environment for the patient The nurse is caring for a patient who becomes nauseated and vomits without warning. The nurse has contaminated hands. Which action is best for the nurse to take next? Wash hands with an antimicrobial soap and water. The nurse is performing hand hygiene before assisting a health care provider with insertion of a chest tube. While washing hands, the nurse touches the sink. Which action will the nurse take next? Repeat handwashing using antiseptic soap The nurse on the surgical team and the surgeon have completed a surgery. After donning gloves, gathering instruments, and placing in the transport carrier, what is the next step in handling the instruments used during the procedure? Sending to central sterile for cleaning and sterilization The nurse is observing a family member changing a dressing for a patient in the home health environment. Which observation indicates the family member has a correct understanding of how to manage contaminated dressings? The family member places the used dressings in a plastic bag The nurse is caring for a group of patients. Which patient will the nurse see first? A patient with Clostridium difficile in droplet precautions The home health nurse is teaching a patient and family about hand hygiene in the home. Which situation will cause the nurse to emphasize washing hands before and after? Performing treatments The surgical mask the perioperative nurse is wearing becomes moist. Which action will the perioperative nurse take next? Apply a new mask The nurse is caring for a patient on contact precautions. Which action will be most appropriate to prevent the spread of disease? Use a dedicated blood pressure cuff that stays in the room and is used for that patient only The nurse is caring for a patient who has cultured positive for Clostridium difficile. Which action will the nurse take next? Instruct assistive personnel to use soap and water rather than sanitizer The nurse is changing linens for a postoperative patient and feels a prick in the left hand. A non-activated safe needle is noted in the linens. For which condition is the nurse most at risk? Hepatitis B The nurse is caring for a patient who has a bloodborne pathogen. The nurse splashes blood above the glove to intact skin while discontinuing an intravenous (IV) infusion. Which step(s) will the nurse take next? Immediately wash the site with soap and running water, and seek guidance from the manager Which process will be required after exposure of a nurse to blood by a cut from a used scalpel in the operative area? Testing the patient and offering treatment to the nurse The nurse is caring for a patient who needs a protective environment. The nurse has provided the care needed and is now leaving the room. In which order will the nurse remove the personal protective equipment, beginning with the first step? Remove gloves. Remove eyewear/face shield and goggles. Untie gown, allow gown to fall from shoulders, and do not touch outside of gown; dispose of properly. Remove mask by strings; do not touch outside of mask. Perform hand hygiene, leave room, and close door. Dispose of all contaminated supplies and equipment in designated receptacles. The nurse manager is evaluating current infection control data for the intensive care unit. The nurse compares past patient data with current data to look for trends. The nurse manager examines the infection chain for possible solutions. In which order will the nurse arrange the items for the infection chain beginning with the first step? An infectious agent or pathogen // Pathogen > Reservoir > Exit > Transmission > Entry > Host A reservoir or source for pathogen growth A portal of exit from the reservoir A mode of transmission A portal of entry to a host A susceptible host The nurse is caring for a patient in protective environment. Which actions will the nurse take? (Select all that apply.) Maintain airflow rate greater than 12 air exchanges/hr Open drapes during the daytime Listen to the patient’s interests The nurse is assessing a new patient admitted to home health. Which questions will be mostappropriate for the nurse to ask to determine the risk of infection? (Select all that apply.) “Can you explain the risk for infection in your home?” “Have you traveled outside of the United States?” “Will you demonstrate how to wash your hands?” “What are the signs and symptoms of infection?” The circulating nurse in the operating room is observing the surgical technologist while applying a sterile gown and gloves to care for a patient having an appendectomy. Which behaviors indicate to the nurse that the procedure by the surgical technologist is correct? (Select all that apply.) Touches only the inside of gown Slips arms into arm holes simultaneously Extended fingers fully into both of the gloves Uses hands covered by sleeves to open gloves The nurse is preparing to insert a urinary catheter. The nurse is using open gloving to apply the sterile gloves. Which steps will the nurse take? (Select all that apply.) With gloved dominant hand, slip fingers underneath second glove cuff Lay glove package on clean flat surface above waistline Glove the dominant hand of the nurse first After second glove is on, interlock hands The nurse has received a report from the emergency department that a patient with tuberculosis will be coming to the unit. Which items will the nurse need to care for this patient? (Select all that apply.) Private room Negative-pressure airflow in room N95 respirator, gown, gloves, eyewear Communication signs for airborne precautions The nurse and the student nurse are caring for two different patients on the medical-surgical unit. One patient is in airborne precautions, and one is in contact precautions. The nurse explains to the student different interventions for care. Which information will the nurse include in the teaching session? (Select all that apply.) Dispose of supplies to prevent the spread of microorganisms. Wash hands before entering and leaving both of the patients’ rooms Apply the knowledge the nurse has of the disease process to prevent the spread of microorganisms Have patients in airborne precautions wear a mask during transportation to other departments Chapter 30 — Vital Signs Apnea—Respirations cease for several seconds. Persistent cessation results in respiratory arrest. Tachypnea—Rate of breathing is regular but abnormally rapid (greater than 20 breaths/min). Kussmaul’s—Respirations are abnormally deep, regular, and increased in rate. Hyperventilation—Rate and depth of respirations increase; breaths are not labored, w/ hypocarbia. Cheyne-Stokes—RR and depth are irregular, characterized by alternating periods of apnea and hyperventilation. Biot’s—Respirations are abnormally shallow for 2 to 3 breaths followed by irregular period of apnea. A patient has a head injury and damages the hypothalamus. Which vital sign will the nurse monitor most closely? Temperature A patient presents with heatstroke. The nurse uses cool packs, cooling blanket, and a fan. Which technique is the nurse using when the fan produces heat loss? Convection The patient has a temperature of 105.2° F. The nurse is attempting to lower temperature by providing tepid sponge baths and placing cool compresses in strategic body locations. Which technique is the nurse using to lower the patient’s temperature? Conduction A nurse is focusing on temperature regulation of newborns and infants. Which action will the nurse take? Place a cap on their heads The nurse is working the night shift on a surgical unit and is making 4:00 AM rounds. The nurse notices that the patient’s temperature is 96.8° F (36° C), whereas at 4:00 PM the preceding day, it was 98.6° F (37° C). What should the nurse do? Realize that this is a normal temperature variation The nurse is caring for a patient who has a temperature reading of 100.4° F (38° C). The patient’s last two temperature readings were 98.6° F (37° C) and 96.8° F (36° C). Which action will the nurse take? Wait 30 minutes and recheck the patient’s temperature A patient is pyrexic. Which piece of equipment will the nurse obtain to monitor this condition? Thermometer The nurse is caring for a patient who has an elevated temperature. Which principle will the nurse consider when planning care for this patient? Hyperthermia occurs when the body cannot reduce heat production The patient with heart failure is restless with a temperature of 102.2° F (39° C). Which action will the nurse take? Place the patient on oxygen The patient requires temperatures to be taken every 2 hours. Which task will the nurse assign to an RN Assessing changes in body temperature The patient requires routine temperature assessment but is confused, easily agitated, and has a history of seizures. Which route will the nurse use to obtain the patient’s temperature? Tympanic The patient is being admitted to the emergency department following a motor vehicle accident. The patient’s jaw is broken with several broken teeth. The patient is ashen, has cool skin, and is diaphoretic. Which route will the nurse use to obtain an accurate temperature reading? Tympanic The nurse is caring for an infant and is obtaining the patient’s vital signs. Which artery will the nurse use to best obtain the infant’s pulse? Brachial The patient is found to be unresponsive and not breathing. Which pulse site will the nurse use? Carotid The nurse needs to obtain a radial pulse from a patient. What must the nurse do to obtain a correct measurement? Place the tips of the first two fingers over the groove along the thumb side of the patient’s wrist The nurse is assessing the patient’s respirations. Which action by the nurse is most appropriate? Obtain without the patient knowing The patient’s blood pressure is 140/60. Which value will the nurse record for the pulse pressure? 80 The nurse reviews the laboratory results for a patient and determines the viscosity of the blood is thick. Which laboratory result did the nurse check? Hematocrit The patient is being admitted to the emergency department with reports of shortness of breath. The patient has had chronic lung disease for many years but still smokes. What will the nurse do? Use oxygen cautiously in this patient. A nurse is reviewing capnography results for adult patients. Which value will cause the nurse to follow up? 50 mm Hg The nurse is caring for a patient who has a pulse rate of 48. His blood pressure is within normal limits. Which finding will help the nurse determine the cause of the patient’s low heart rate? The patient has calcium channel blockers or digitalis medication prescriptions The patient was found unresponsive in an apartment and is being brought to the emergency department. The patient has arm, hand, and leg edema, temperature is 95.6° F, and hands are cold secondary to a history of peripheral vascular disease. It is reported that the patient has a latex allergy. What should the nurse do to quickly measure the patient’s oxygen saturation? Place a nonadhesive sensor on the patient’s earlobe The patient is admitted with shortness of breath and chest discomfort. Which laboratory value could account for the patient’s symptoms? Hemoglobin level of 8.0 g/100 mL A nurse reviews blood pressures of several patients. Which finding will the nurse report as prehypertension? 120/80 in a middle-aged adult The nurse is providing a blood pressure clinic for the community. Which group will the nurse most likely address? African-Americans A nurse is caring for a patient who smokes and drinks caffeine. Which point is important for the nurse to understand before assessing the patient’s blood pressure (BP)? Smoking result in vasoconstriction, falsely elevating BP When taking the pulse of an infant, the nurse notices that the rate is 145 beats/min and the rhythm is regular. How should the nurse interpret this finding? This is normal for an infant The nurse is caring for an older-adult patient and notes that the temperature is 96.8° F (36° C). How will the nurse interpret this finding? The patient has a normal temperature When assessing the temperature of newborns and children, the nurse decides to utilize a temporal artery thermometer. What is the rationale for the nurse’s action? It has no risk of injury to patient or nurse The nurse is caring for a small child and needs to obtain vital signs. Which site choice from the nursing assistive personnel (NAP) will cause the nurse to praise the NAP? Brachial site The nurse is caring for a newborn infant in the hospital nursery and notices that the infant is breathing rapidly but is pink, warm, and dry. Which normal respiratory rate will the nurse consider when planning care for this newborn? 30 to 60 The nurse is preparing to obtain an oxygen saturation reading on a toddler. Which action will the nurse take? Determine whether the toddler has a latex allergy The nurse is preparing to assess the blood pressure of a 3-year-old. How should the nurse proceed? Explain the procedure to the child A nurse is caring for a group of patients. Which patient will the nurse see first? A calm adolescent with P-95 and R-26 The nurse is caring for a patient who is being discharged from the hospital after being treated for hypertension. The patient is instructed to take blood pressure 3 times a day and to keep a record of the readings. The nurse recommends that the patient purchase a portable electronic blood pressure device. Which other information will the nurse share with the patient? You will need to recalibrate the machine The nurse is caring for a patient who reports feeling light-headed and “woozy.” The nurse checks the patient’s pulse and finds that it is irregular. The patient’s blood pressure is 100/72. It was 113/80 an hour earlier. What should the nurse do? Perform an apical/radial pulse assessment A nurse is caring for a group of patients. Which patient will the nurse see first? A 20-year-old male postoperative patient whose blood pressure went from 128/70 to 100/60 The health care provider prescription reads “Metoprolol (Lopressor) 50 mg PO daily. Do not give if blood pressure is less than 100 mm Hg systolic.” The patient’s blood pressure is 92/66. The nurse does not give the medication. Which action should the nurse take? Documents that the medication was not given because of low blood pressure After taking the patient’s temperature, the nurse documents the value and the route used to obtain the reading. What is the reason for the nurse’s action? Temperatures vary depending on the route used When taking an adult blood pressure, the onset of the sound the nurse hears is at 138, the muffled sound the nurse hears is at 70, and the disappearance of the sound the nurse hears is at 62. How should the nurse record this finding? 138/62 The nursing assistive personnel (NAP) is taking vital signs and reports that a patient’s blood pressure is abnormally low. What should the nurse do next? Retake the blood pressure personally and assess the patient’s condition A nurse is working in the intensive care unit and must obtain core temperatures on patients. Which sites can be used to obtain a core temperature? (Select all that apply.) Tympanic Esophagus Pulmonary artery The patient has new-onset restlessness and confusion. Pulse rate is elevated, as is respiratory rate. Oxygen saturation is 94%. The nurse ignores the pulse oximeter reading and calls the health care provider for orders because the pulse oximetry reading is inaccurate. Which factors can cause inaccurate pulse oximetry readings? (Select all that apply.) Carbon monoxide inhalation Hypothermic fingers Intravascular dyes Nail polish Jaundice The nurse is assessing the patient and family for probable familial causes of the patient’s hypertension. The nurse begins by analyzing the patient’s personal history, as well as family history and current lifestyle situation. Which findings will the nurse consider to be risk factors? (Select all that apply.) Obesity Cigarette smoking Heavy alcohol intake The patient is being encouraged to purchase a portable automatic blood pressure device to monitor blood pressure at home. Which information will the nurse present as benefits for this type of treatment? (Select all that apply.) Patients can actively participate in their treatment Self-monitoring helps with compliance and treatment Patients can provide information about patterns to health care providers A nurse is teaching the staff about alterations in breathing patterns. Which information will the nurse include in the teaching session? (Select all that apply.) Apnea—no respirations Tachypnea—regular, rapid respirations Kussmaul’s—abnormally deep, regular, fast respirations A nurse is assessing results of vital signs for a group of patients. Match the condition to the assessment findings the nurse is reviewing. Hypothermia – Patient’s temperature is 93.2° F (34° C). Shock/Hypotension - Patient’s blood pressure went from 126/76 to 90/50 Heatstroke - Patient’s temperature is 113° F (45° C) with hot, dry skin Orthostatic hypotension - Patient’s blood pressure sitting is 130/60 and 110/40 standing Tachycardia - Patient’s pulse is 110 beats/min Chapter 31 — Health Assessment and Physical A nurse is a preceptor for a nurse who just graduated from nursing school. When caring for a patient, the new graduate nurse begins to explain to the patient the purpose of completing a physical assessment. Which statement made by the new graduate nurse requires the preceptor to intervene? “Nursing assessment data are used only to provide information about the effectiveness of your medical care.” Having misplaced a stethoscope, a nurse borrows a colleague’s stethoscope. The nurse next enters the patient’s room and identifies self, washes hands with soap, and states the purpose of the visit. The nurse performs proper identification of the patient before auscultating the patient’s lungs. Which critical health assessment step should the nurse have performed? Cleaning stethoscope with alcohol A nurse is preparing to perform a complete physical examination on a weak, older-adult patient with bilateral basilar pneumonia. Which position will the nurse use? Supine A nurse is conducting Weber’s test. Which action will the nurse take? Place a vibrating tuning fork in the middle of patient’s forehead. A head and neck physical examination is completed on a 50-year-old female patient. All physical findings are normal except for fine brittle hair. Which laboratory test will the nurse expect to be ordered, based upon the physical findings? Thyroid-stimulating hormone test A febrile preschool-aged child presents to the after-hours clinic. Varicella (chickenpox) is diagnosed on the basis of the illness history and the presence of small, circumscribed skin lesions filled with serous fluid. Which type of skin lesion will the nurse report? Vesicles A school nurse recognizes a belt buckle–shaped ecchymosis on a 7-year-old student. When privately asked about how the injury occurred, the student described falling on the playground. Which action will the nurse take next? Contact social services and report suspected abuse. A nurse identifies lice during a child’s scalp assessment. The nurse teaches the parents about hair care. Which information from the parents indicates the nurse needs to follow up? We will use lindane-based shampoos. A parent calls the school nurse with questions regarding the recent school vision screening. Snellen chart examination revealed 20/60 for both eyes. Which response by the nurse is the best regarding the eye examination results? Your child needs to see an ophthalmologist. During a routine pediatric history and physical, the parents report that their child was a very small, premature infant that had to stay in the neonatal intensive care unit longer than usual. They state that the infant was yellow when born and developed an infection that required “every antibiotic under the sun” to reach a cure. Which exam is a priority for the nurse to conduct on the child? Hearing acuity During a sexually transmitted illness presentation to high-school students, the nurse recommends the human papillomavirus (HPV) vaccine series. Which condition is the nurse trying to prevent? Cervical cancer A male student comes to the college health clinic. He hesitantly describes that he found something wrong with his testis when taking a shower. Which assessment finding will alert the nurse to possible testicular cancer? Hard, pea-sized testicular lump The nurse is urgently called to the gymnasium regarding an injured student. The student is crying in severe pain with a malformed fractured lower leg. Which proper sequence will the nurse follow to perform the initial assessment? Inspection and light palpation The nurse is examining a female with vaginal discharge. Which position will the nurse place the patient for proper examination? Lithotomy On admission, a patient weighs 250 pounds. The weight is recorded as 256 pounds on the second inpatient day. Which condition will the nurse assess for in this patient? Fluid retention The patient is a 45-year-old African-American male who has come in for a routine annual physical. Which type of preventive screening does the nurse discuss with the patient? Digital rectal examination of the prostate An advanced practice nurse is preparing to assess the external genitalia of a 25-year-old American woman of Chinese descent. Which action will the nurse do first? Assess the patient’s feelings about the examination. An older-adult patient is being seen for chronic entropion. Which condition will the nurse assess for in this patient? Infection During a school physical examination, the nurse reviews the patient’s current medical history. The nurse discovers the patient has allergies. Which assessment finding is consistent with allergies? Pale nasal mucosa Upon assessment, the patient is breathing normally and has normal vesicular lung sounds. Which expected inspiratory-to-expiratory breath sounds will the nurse hear? The inspiratory phase is 3 times longer than the expiratory phase. A nurse is conducting Weber’s test. Which action will the nurse take? Place a vibrating tuning fork in the middle of patient’s forehead. A head and neck physical examination is completed on a 50-year-old female patient. All physical findings are normal except for fine brittle hair. Which laboratory test will the nurse expect to be ordered, based upon the physical findings? Thyroid-stimulating hormone test A febrile preschool-aged child presents to the after-hours clinic. Varicella (chickenpox) is diagnosed on the basis of the illness history and the presence of small, circumscribed skin lesions filled with serous fluid. Which type of skin lesion will the nurse report? Vesicles A school nurse recognizes a belt buckle–shaped ecchymosis on a 7-year-old student. When privately asked about how the injury occurred, the student described falling on the playground. Which action will the nurse take next? Contact social services and report suspected abuse. A nurse identifies lice during a child’s scalp assessment. The nurse teaches the parents about hair care. Which information from the parents indicates the nurse needs to follow up? We will use lindane-based shampoos. A parent calls the school nurse with questions regarding the recent school vision screening. Snellen chart examination revealed 20/60 for both eyes. Which response by the nurse is the best regarding the eye examination results? Your child needs to see an ophthalmologist. During a routine pediatric history and physical, the parents report that their child was a very small, premature infant that had to stay in the neonatal intensive care unit longer than usual. They state that the infant was yellow when born and developed an infection that required “every antibiotic under the sun” to reach a cure. Which exam is a priority for the nurse to conduct on the child? Hearing acuity During a sexually transmitted illness presentation to high-school students, the nurse recommends the human papillomavirus (HPV) vaccine series. Which condition is the nurse trying to prevent? Cervical cancer A male student comes to the college health clinic. He hesitantly describes that he found something wrong with his testis when taking a shower. Which assessment finding will alert the nurse to possible testicular cancer? Hard, pea-sized testicular lump The nurse is urgently called to the gymnasium regarding an injured student. The student is crying in severe pain with a malformed fractured lower leg. Which proper sequence will the nurse follow to perform the initial assessment? Inspection and light palpation The nurse is examining a female with vaginal discharge. Which position will the nurse place the patient for proper examination? Lithotomy On admission, a patient weighs 250 pounds. The weight is recorded as 256 pounds on the second inpatient day. Which condition will the nurse assess for in this patient? Fluid retention The patient is a 45-year-old African-American male who has come in for a routine annual physical. Which type of preventive screening does the nurse discuss with the patient? Digital rectal examination of the prostate An advanced practice nurse is preparing to assess the external genitalia of a 25-year-old American woman of Chinese descent. Which action will the nurse do first? Assess the patient’s feelings about the examination. An older-adult patient is being seen for chronic entropion. Which condition will the nurse assess for in this patient? Infection During a school physical examination, the nurse reviews the patient’s current medical history. The nurse discovers the patient has allergies. Which assessment finding is consistent with allergies? Pale nasal mucosa Upon assessment, the patient is breathing normally and has normal vesicular lung sounds. Which expected inspiratory-to-expiratory breath sounds will the nurse hear? The inspiratory phase is 3 times longer than the expiratory phase. A teen female patient reports intermittent abdominal pain for 12 hours. No dysuria is present. Which action will the nurse take when performing an abdominal assessment? Ask the patient about the color of her stools. During a genitourinary examination of a 30-year-old male patient, the nurse identifies a small amount of a white, thick substance on the patient’s uncircumcised glans penis. What is the nurse’s next step? Record this as a normal finding. The nurse is preparing for a rectal examination of a nonambulatory male patient. In which position will the nurse place the patient? Sims’ A teen patient is tearful and reports locating lumps in her breasts. Other history obtained is that she is currently menstruating. Physical examination reveals soft and movable cysts in both breasts that are painful to palpation. The nurse also notes that the patient’s nipples are erect, but the areola is wrinkled. Which action will the nurse take after talking with the health care provider? Discuss the possibility of fibrocystic disease as the probable cause. A nurse is performing a mental status examination and asks an adult patient what the statement “Don’t cry over spilled milk” means. Which area is the nurse assessing? Abstract thinking During a routine physical examination of a 70-year-old patient, a blowing sound is auscultated over the carotid artery. Which assessment finding will the nurse report to the health care provider? Bruit The nurse considers several new female patients to receive additional teaching on the need for more frequent Pap test and gynecological examinations. Which assessment findings reveal the patient at highest risk for cervical cancer and having the greatest need for patient education? 22 years old, smokes 1 pack of cigarettes per day, has multiple sexual partners The paramedics transport an adult involved in a motor vehicle accident to the emergency department. On physical examination, the patient’s level of consciousness is reported as opening eyes to pain and responding with inappropriate words and flexion withdrawal to painful stimuli. Which value will the nurse report for the patient’s Glasgow Coma Scale score? 9 While assessing the skin of an 82-year-old patient, a nurse discovers nonpainful, ruby red papules on the patient’s trunk. What is the nurse’s next action? Document cherry angiomas as a normal older adult skin finding. A nurse is caring for a group of patients. Which patient will the nurse see first? An adult with an S4 heart sound A nurse is auscultating different areas on an adult patient. Which technique should the nurse use during an assessment? Uses the diaphragm to listen for bowel sounds The nurse is assessing an adult patient’s patellar reflex. Which finding will the nurse record as normal? 2+ A patient in the emergency department is reporting left lower abdominal pain. Which proper order will the nurse follow to perform the comprehensive abdominal examination? Inspection, auscultation, palpation The nurse completed assessments on several patients. Which assessment finding will the nurse record as normal? Constricting pupils when directly illuminated The patient presents to the clinic with dysuria and hematuria. How does the nurse proceed to assess for kidney inflammation? Percusses posteriorly the costovertebral angle at the scapular line. The nurse is assessing skin turgor. Which technique will the nurse use? Grasp a fold of skin on the sternal area. An older-adult patient is taking aminoglycoside for a severe infection. Which assessment is the priority? Ears The patient has had a stroke that has affected the ability to speak. The patient becomes extremely frustrated when trying to speak. The patient responds correctly to questions and instructions but cannot form words coherently. Which type of aphasia is the patient experiencing? Expressive The nurse is assessing the tympanic membranes of an infant. Which action by the nurse demonstrates proper technique? Uses an inverted otoscope grip while pulling the auricle downward and back. A nurse is assessing a patient’s cranial nerve IX. Which items does the nurse gather before conducting the assessment? (Select all that apply.) Vial of sugar, Tongue blade, Lemon applicator A nurse is assessing several patients. Which assessment findings will cause the nurse to follow up? (Select all that apply.) Orthopnea Pleural friction rub present Crackles in lower lung lobes A nurse is preparing to perform a lung assessment on a patient and discovers through the nursing history the patient smokes. The nurse figures the pack-years for this patient who has smoked two and a half (2 1/2) packs a day for 20 years. Which value will the nurse record in the patient’s medical record? Record answer as a whole number. pack-years 50 A nurse is assessing a group of patients. Match the assessment finding the nurse observed to its condition. Koilonychia - Spoon nails Venous problems - Lower extremity swollen and warm with normal pulse Lordosis - Swayback Melena - Black, tarry stools Arterial problems - Lower extremity pale and cool with decreased pulse Jugular vein distention - Neck vein visible when sitting Tinnitus - Ringing in ears Chapter 32 — Medication Administration A nurse is teaching a patient about medications. Which statement from the patient indicates teaching is effective? “I will rotate the sites in my left leg when I give my insulin.” A nurse is preparing to administer an injection to a patient. Which statement made by the patient is an indication for the nurse to use the Z-track method? “The last shot like that turned my skin colors.” A 2-year-old child is ordered to have eardrops daily. Which action will the nurse take? Pull the auricle down and back to straighten the ear canal. A patient has an order to receive 0.3 mL of U-500 insulin. Which syringe will the nurse obtain to administer the medication? Tuberculin syringe A patient has an order to receive 12.5 mg of hydrochlorothiazide. The nurse has on hand a 25 mg tablet of hydrochlorothiazide. How many tablet(s) will the nurse administer? 1/2 Tablet The patient is to receive phenytoin (Dilantin) at 0900. When will be the ideal time for the nurse to schedule a trough level? 0830 A patient is receiving vancomycin. Which function is the priority for the nurses to assess? Hearing A health care provider orders lorazepam (Ativan) 1 mg orally 2 times a day. The dose available is 0.5 mg per tablet. How many tablet(s) will the nurse administer for each dose? 2 The nurse is preparing to administer an injection into the deltoid muscle of an adult patient. Which needle size and length will the nurse choose? 25 gauge × 1 inch When the nurse administers an IM corticosteroid injection, the nurse aspirates. What is the rationale for the nurse aspirating? Ensure proper placement of the needle. The nurse is giving an IM injection. Upon aspiration, the nurse notices blood return in the syringe. What should the nurse do? Withdraw the needle and prepare the injection again. The nurse is planning to administer a tuberculin test with a 27-gauge, -inch needle. At which angle will the nurse insert the needle? 15 degree The nurse closely monitors an older adult for signs of medication toxicity. Which physiological change is the reason for the nurse’s action? Reduced glomerular filtration A registered nurse interprets that a scribbled medication order reads 25 mg. The nurse administers 25 mg of the medication to a patient and then discovers that the dose was incorrectly interpreted and should have been 15 mg. Who is ultimately responsible for the error? Nurse A patient is to receive a proton pump inhibitor through a nasogastric (NG) feeding tube. Which is the most important nursing action to ensure effective absorption? Hold feeding for at least 30 minutes after medication administration. A health care provider prescribes aspirin 650 mg every 4 hours PO when febrile. For which patient will this order be appropriate? 62 year old with a high fever from an infection A patient is in need of immediate pain relief for a severe headache. Which medication will the nurse administer to be absorbed the quickest? Hydromorphone 4 mg IV While preparing medications, the nurse knows one of the drug is an acidic medication. In which area does the nurse anticipate the drug will be absorbed? Stomach The nurse administers a central nervous system stimulant to a patient. Which assessment finding indicates to the nurse that an idiosyncratic event is occurring? Falls asleep during daily activities An order is written for phenytoin 500 mg IM q3-4h prn for pain. The nurse recognizes that treatment of pain is not a standard therapeutic indication for this drug. The nurse believes that the health care provider meant to write hydromorphone. What should the nurse do? Call the health care provider to clarify the order. A patient needs assistance in eliminating an anesthetic gaseous medication (nitrous oxide). Which action will the nurse take? Encourage the patient to cough and deep-breathe. A nurse has withdrawn a narcotic from the medication dispenser and must waste a portion of the medication. What should the nurse do? Have another nurse witness the wasted medication. A nurse teaches the patient about the prescribed buccal medication. Which statement by the patient indicates teaching by the nurse is successful? “I should let the medication dissolve completely.” What is the nurse’s priority action to protect a patient from medication error? Reading medication labels at least 3 times before administering The nurse prepares a pain injection for a patient but had to check on another patient and asks a new nurse to give the medication. Which action by the new nurse is best? Do not give the medication. A patient is at risk for aspiration. Which nursing action is most appropriate? Have the patient self-administer the medication. A patient refuses medication. Which is the nurse’s first action? Explore with the patient reasons for not wanting to take the medication. A patient who is being discharged today is going home with an inhaler. The patient is to administer 2 puffs of the inhaler twice daily. The inhaler contains 200 puffs. When should the nurse appropriately advise the patient to refill the medication? 6 weeks from the start of using the inhaler The supervising nurse is watching nurses prepare medications. Which action by one of the nurses will the supervising nurse stop immediately? Prepares NPH insulin to be given intravenously (IV) Which patient does the nurse most closely monitor for an unintended synergistic effect? The 72 year old who is seeing four different specialists Which patient using an inhaler would benefit most from using a spacer? A 25 year old with limited coordination of the extremities The prescriber wrote for a 40-kg child to receive 25 mg of medication 4 times a day. The therapeutic range is 5 to 10 mg/kg/day. What is the nurse’s priority? Notify the health care provider that the prescribed dose is below the therapeutic range. The supervising nurse is observing several different nurses. Which action will cause the supervising nurse to intervene? A nurse draws up the NPH insulin first when mixing a short-acting and intermediate-acting insulin. A nurse is caring for a patient who is receiving pain medication through a saline lock. After obtaining a good blood return when the nurse is flushing the patient’s peripheral IV, the patient reports pain. Upon assessment, the nurse notices a red streak that is warm and tender to the touch. What is the nurse’s initial action? Do not administer the pain medication. The nurse is preparing to administer medications to two patients with the same last name. After the administration, the nurse realizes that did not check the identification of the patient before administering medication. Which action should the nurse complete first? Return to the room to check and assess the patient. The nurse is caring for two patients with the same last name. In this situation which right of medication administration is the priority to reduce the chance of an error? Right patient A patient prefers not to take the daily allergy pill this morning because it causes drowsiness throughout the day. Which response by the nurse is best? “Let’s see if we can change the time you take your pill to 9 PM, so the drowsiness occurs when you would normally be sleeping.” A nurse is preparing to administer a medication from a vial. In which order will the nurse perform the steps, starting with the first step? Clean with alcohol swab and allow to dry. // Clean > Pull > Inject > Invert > Fill > Tap Pull back on the plunger the amount to be drawn up. Inject air into the airspace of the vial. Invert the vial. Fill the syringe with medication. Tap the side of the syringe barrel to remove air bubbles. A nurse is attempting to administer an oral medication to a child, but the child refuses to take the medication. A parent is in the room. Which statement by the nurse to the parent is best? “I will prepare the medication for you and observe if you would like to try to administer the medication.” An older-adult patient needs an IM injection of antibiotic. Which site is best for the nurse to use? Ventrogluteal A nurse is preparing an intravenous IV piggyback infusion. In which order will the nurse perform the steps, starting with the first one? Compare the label of the medication with the medication administration record at the patient’s bedside. Connect the infusion tubing to the medication bag. Hang the piggyback medication bag above the level of the primary fluid bag. Clean the main IV line port with an antiseptic swab. Connect the tubing of the piggyback infusion to the appropriate connector on the upper Y-port. Regulate flow. Compare > Connect infusion > Hang > Clean > Connect > Regulate A nurse is administering oral medications to patients. Which action will the nurse take? Place all of the patient’s medications in the same cup, except medications with assessments. A nurse is performing the three accuracy checks before administering an oral liquid medication to a patient. When will the nurse perform the second accuracy check? Before going to the patient’s room A nurse is preparing to administer an antibiotic medication at 1000 to a patient but gets busy in another room. When should the nurse give the antibiotic medication? By 1030 The nurse is administering medications to several patients. Which action should the nurse take? Advise a patient after a corticosteroid inhaler treatment to rinse mouth with water. A nurse is following safety principles to reduce the risk of needlestick injury. Which actions will the nurse take? (Select all that apply.) Never force needles into the sharps disposal. Use clearly marked sharps disposal containers. Use needleless devices whenever possible. Which methods will the nurse use to administer an intravenous (IV) medication that is incompatible with the patient’s IV fluid? (Select all that apply.) Start another IV site. Flush with 10 mL of sterile water before and after administration. Flush with 10 mL of normal saline before and after administration. A patient is taking 1 tablet of hydrocodone bitartrate 5 mg and acetaminophen 500 mg (Vicodin) every 4 hours. The patient is also taking 2 tablets of acetaminophen (Tylenol) 325 mg every 12 hours. How many grams of acetaminophen is the patient taking daily? Record your answer using one decimal place. g 3.3 The nurse is administering 250 mg of a medication elixir to the patient. The medication comes in a dose of 1000 mg/5 mL. How many milliliters should the nurse administer? Record your answer using two decimal places. mL 1.25 The patient is to receive amoxicillin 500 mg q8h; the medication is dispensed at 250 mg/5 mL. How many milliliters will the nurse administer for one dose? Record your answer using a whole number. mL 10 Chapter 33—Complementary and Alternative Therapies A patient describes practicing a complementary and alternative therapy involving breathwork and yoga. The nurse also recommends using energy field therapies. Which techniques did the nurse suggest? Reiki therapy and therapeutic touch A teen with an anxiety disorder is referred for biofeedback because the parents do not want their child to take anxiolytics. Which statement from the teen indicates successful learning? “Biofeedback will help me with my thoughts and physiological responses to stress.” An older-adult patient is newly admitted to a skilled nursing facility with the diagnoses of Alzheimer’s dementia, lipidemia, and hypertension, and a history of pulmonary embolism. Medications brought on admission included lisinopril (Zestril, Prinivil), hydrochlorothiazide (Microzide), warfarin (Coumadin), low-dose aspirin, ginkgo biloba, and echinacea. Which potential interaction will cause the nurse to notify the patient’s health care provider? Warfarin and ginkgo biloba A patient asks the nurse for a nonmedical approach for excessive worry and work stress. Which therapy should the nurse recommend? Meditation A nurse is teaching about the therapy that is more effective in treating physical ailments than in preventing disease or managing chronic illness. Which therapy is the nurse describing? Allopathic During a relaxation therapy skills group, the instructor discusses the cognitive skill of learning to tolerate uncertain and unfamiliar experiences. Which skill is the nurse describing? Receptivity A patient asks about the new clinic in town that is staffed by allopathic and complementary practitioners. Which response from the nurse is best? It is probably an integrative medical clinic. The group leader is overheard saying to the gathering of patients, “Focus on your breathing once again …. Notice how it is regular …. Now focus on your left arm …. Notice how relaxed your left arm feels …. Notice the relaxation going down the left arm to the hand.” A patient asks the nurse what the group is doing. What is the nurse’s best response? It is progressive relaxation training. A therapeutic touch practitioner scans the patient’s body. What is the purpose of the practitioner’s actions? To identify energy obstructions A nurse is teaching a patient about meridians. Which technique is the nurse preparing the patient to receive? Acupuncture A Native American patient is asking for a spiritual healer. Which person should the nurse try to contact for the patient? Shaman A nurse is using the holistic approach to care. Which goal is the priority? Incorporate the mind-body-spirit connection. A nurse is using caring-healing relationships to support whole person/whole systems healing. Which type of nursing is the nurse using? Integrative nursing A nurse is teaching a patient relaxation techniques to decreases stress. Which finding will support the nurse’s evaluation that the therapy is effective? Decreased heart rate A nurse is providing different types of therapies to a patient with excessive fatigue and cancer. Which technique will cause the nurse manager to intervene? Active progressive relaxation A nurse is emphasizing the use of touch to decrease “skin hunger” in caring for patients. Which age group is the nurse primarily describing? Older adults A patient is proficient at meditation from long-time use of the technique. Which finding in the medication history will cause the nurse to follow up? Takes thyroid-regulating medication A patient is taking an antidepressant medication. The nurse discovers that the patient uses herbs. Which herb will cause the nurse to intervene? Chamomile A nurse is teaching a patient about the use of biofeedback. Which goal should the nurse add to the care plan? Controls autonomic physiological functions Which patient will cause the nurse to question an order for acupuncture? A patient with AIDS A nurse is describing the therapeutic effects of imagery. Which information should the nurse include in the teaching session? (Select all that apply.) Controls pain Decreases nightmares Helps with irritable bowel syndrome A nurse is teaching a class over herbs and their uses. Which condition will the nurse match to its specific herb for the teaching session? Aloe - Skin inflammation Garlic - Elevated cholesterol levels Valerian - Mild anxiety Ginger - Nausea and vomiting Saw palmetto - Benign prostatic hyperplasia Chaparral - Unsafe and should not be used Chapter 34 — Self-Concept The nurse is caring for an older-adult patient. Which technique will the nurse use to enhance an older-adult patient’s self-concept? Reviewing old photos with patients While gathering an adolescent’s health history, the nurse recognizes that the patient began to act out behaviorally and engaged in risky behavior when the patient’s parents divorced. Which information will the nurse gather to determine situational low self-esteem? How the patient views behaviors A nurse is assessing a group of adolescents. Which person is most likely to have the highest self-esteem? Latino adolescent female who has strong ethnic pride A nurse is caring for a patient who is dealing with the developmental task known as initiative versus guilt. The nurse is providing care to which patient? A 5-year-old kindergarten student A verbally abusive partner has told a significant other many negative comments over the years. In the crisis center, the nurse would anticipate that the patient may have which self-concept deficits? Body image Two 50-year-old men are discussing their Saturday activities. The first man describes how he tutors children as a volunteer at a community center. The other man says that he would never work with children and that he prefers to work out at the gym to meet young women to date. Which developmental stage is the second man exhibiting? Self-absorption An adult son is adjusting to the idea of his chronically ill parents moving into his family home. The community health nurse is assessing the adult son for potential stressors secondary to the new family living arrangement. Which stressor will the nurse assess for in this adult son? Role overload A nurse grimaces while changing a patient’s colostomy bag. Which effect will the nurse’s behavior most likely have on the patient? Promote development of a negative body image. A male patient states, “I’m such a loser. I only had that job for a month.” Which outcome criteria will the nurse add to the patient’s care plan? The patient will verbalize two life areas in which he functions well. A nurse is teaching a patient about self-concept. Which information from the patient indicates the nurse needs to follow up about components of self-concept? One component is coping. Which individual is most likely to need the nurse’s assistance in coping with identity confusion? A 35-year-old recently divorced mother of twins A nurse is assessing a patient for possible altered self-concept. Which assessment finding is consistent with altered self-concept? Hesitant to express opinions A nurse is caring for a postoperative mastectomy patient. Which action is a priority for increasing self-awareness? Using communication skills to clarify family and patient expectations The nurse in an addictions clinic is working with a patient on priority setting before the patient’s discharge from residential treatment. Which goal is a priority at this time? Identifying local self-help groups before being discharged from the program A nurse is teaching a patient about self-concept. Which information from the patient indicates a correct understanding of the teaching? Self-concept is how a person thinks about oneself. A nurse is evaluating a patient’s self-concept. Which key indicator will the nurse use? Nonverbal behavior A nurse is assessing a patient’s self-concept. Which area should the nurse assess first? Role performance A 9-year-old is proudly telling the nurse about mastering the yellow belt in a martial arts class. Which developmental stage is the child exhibiting? Industry versus inferiority A nurse is developing a drinking prevention presentation for adolescents. Which areas should the nurse include in the teaching session? Stress management and improving self-esteem A nurse is completing a history on a patient with role conflict. Which finding is consistent with role conflict? A patient has to travel for work and misses children’s birthdays. A nurse is assessing a patient’s self-concept. Which areas will the nurse include? (Select all that apply.) Identity Body image Coping behaviors Significant others’ support A nurse is assessing a group of patients. Match the assessment finding to the area the nurse is assessing. Identity confusion - I am one with the universe Disturbed body image - I am ugly with all these burn scars Role performance - I am a good mother Low self-esteem - I am good for nothing Chapter 36—Spiritual Health A co-worker asks the nurse to explain spirituality. What is the nurse’s best response? It is awareness of one’s inner self. The nurse is caring for a patient who is an agnostic. Which information should the nurse consider when planning care for this patient? The patient believes there is no known ultimate reality. The nurse is caring for an Islamic patient who wants a snack. Which action by the nurse is most appropriate? Offers a beef sandwich A nurse is teaching a patient how to meditate. Which information from the patient indicates effective learning? I will focus on an image. The nurse is admitting a patient to the hospital. The patient is a very spiritual person but does not practice any specific religion. How will the nurse interpret this finding? This statement is reasonable. A nurse hears the following comments from different patients. Which patient comment does the nurse identify as faith? I believe there is life after death. Allow time for purity rituals. The nurse is caring for a patient with a chronic illness who is having conflicts with beliefs. Which health care team member will the nurse ask to see this patient? The clergy The nurse is caring for a patient with a terminal disease. The nurse sits down and lightly touches the patient’s hand. Which technique is the nurse using? Establishing presence The nurse and the patient have the same religious affiliation. Which action will the nurse take? Do not impose personal values on the patient Interpersonal The patient is admitted with chronic anxiety. Which action is most appropriate for the nurse to take? Look at how anxiety influences the patient’s ability to function. In caring for the patient’s spiritual needs, the nurse asks 20 questions to assess the patient’s relationship with God and a sense of life purpose and satisfaction. Which method is the nurse using? The spiritual well-being scale Allow the wife and daughters to visit at the patient’s request The nurse is caring for a patient who has been diagnosed with a terminal illness. The patient states, “I just don’t feel like going to work. I have no energy, and I can’t eat or sleep.” The patient shows no interest in taking part in the care by saying, “What’s the use?” Which response by the nurse is best? It sounds like you have lost hope. Use therapeutic communication to establish trust and caring. The nurse is caring for a group of patients. Which patient will the nurse see first? A patient saying that God has left and there is no reason for living. A nurse is providing spiritual care to patients. Which action is essential for the nurse to take? Know one’s own personal beliefs. Review the patient’s view of the purpose in life. Ask whether the patient’s expectations were met. Discuss with family and friends the patient’s connectedness. Review the patient’s self-perception regarding spiritual health. Spiritual distress has been identified in a patient who has been diagnosed with a chronic illness. Which interventions will the nurse add to the care plan? (Select all that apply.) Offer to pray with the patient. Develop activities to heal body, mind, and spirit. Teach relaxation, guided imagery, and meditation A nurse is providing spiritual care to a group of patients. Match the group to their belief. Nature controls life and health. Appalachians Organ transplantation or donation is not considered. Islam Observance of the Sabbath is important. Judaism Past sins cause illness. Hinduism Nonhuman spirits invading the body cause illness. Buddhism Chapter 37 – The Experience of Loss, Death, and Grief A nurse encounters a family who experienced the death of their adult child last year. The parents are talking about the upcoming anniversary of their child’s death. The nurse spends time with them discussing their child’s life and death. Which nursing principle does the nurse’s action best demonstrate? Facilitation of normal mourning A cancer patient asks the nurse what the criteria are for hospice care. Which information should the nurse share with the patient? It is for those expected to live less than 6 months. A terminally ill patient is experiencing constipation secondary to pain medication. Which is the best method for the nurse to improve the patient’s constipation problem? Use a laxative. A severely depressed patient cannot state any positive attributes to life. The nurse patiently sits with this patient and assists the patient to identify several activities the patient is actually looking forward to in life. Which spiritual concept is the nurse trying to promote? Hope In preparation for the eventual death of a female hospice patient of the Muslim faith, the nurse organizes a meeting of all hospice caregivers. A plan of care to be followed when this patient dies is prepared. Which information will be included in the plan? Allow female Muslims to care for the body after death has occurred. Family members gather in the emergency department after learning that a family member was involved in a motor vehicle accident. After learning of the family member’s unexpected death, the surviving family members begin to cry and scream in despair. Which phase does the nurse determine the family is in according to the Attachment Theory? Yearning and searching A nursing assistive personnel (NAP) is caring for a dying patient. Which action by the NAP will cause the nurse to intervene? Making the patient eat An Orthodox Jewish rabbi has been pronounced dead. The nursing assistive personnel respectfully ask family members to leave the room and go home as postmortem care is provided. Which statement from the supervising nurse is best? “Family members stay with the body until burial the next day.” A palliative team is caring for a dying patient in severe pain. Which action is the priority? Enhance the patient’s quality of life. A veteran is hospitalized after surgical amputation of both lower extremities owing to injuries sustained during military service. Which type of loss will the nurse focus the plan of care on for this patient? Situational loss “I know it seems strange, but I feel guilty being pregnant after the death of my son last year,” said a woman during her routine obstetrical examination. The nurse spends extra time with this woman, helping her realize bonding with this unborn child will not mean she is replacing the one who died. Which nursing technique does this demonstrate? Facilitating mourning A patient has had two family members die during the past 2 days. Which coping strategy is most appropriate for the nurse to suggest to the patient? Writing in a journal A female nurse is called into the supervisor’s office regarding her deteriorating work performance since the loss of her spouse 2 years ago. The woman begins sobbing and says that she is “falling apart” at home as well. Which type of grief is the female nurse experiencing? Complicated grief A nurse is caring for a patient in the last stages of dying. Which finding indicates the nurse needs to prepare the family for death? Cheyne-Stokes breathing The mother of a child who died recently keeps the child’s room intact. Family members are encouraging her to redecorate and move forward in life. Which type of grief will the home health nurse recognize the mother is experiencing? Normal A nurse is caring for a dying patient. One of the nurse’s goals is to promote dignity and validation of the dying person’s life. Which action will the nurse take to best achieve this goal? Listen to family stories about the person. A nurse is caring for a dying patient. When is the best time for the nurse to discuss end-of-life care? During assessment A nurse is providing postmortem care. Which action will the nurse take? Leave dentures in the mouth. A nurse lets the transplant coordinator make a request for organ and tissue donation from the patient’s family. What is the primary rationale for the nurse’s action? The nurse is following a federal law. A patient cancels a scheduled appointment because the patient will be attending a Shivah for a family member. Which response by the nurse is best? “I’m so sorry for your loss.” During a follow-up visit, a female patient is describing new onset of marital discord with her terminally ill spouse to the hospice nurse. Which Kübler-Ross stage of dying is the patient experiencing? Anger A previously toilet trained toddler has started wetting again. A nurse is gathering a health history from the grandparent. Which health history finding will the nurse most likely consider as the cause of the wetting? Recent parental death A patient’s father died a week ago. Both the patient and the patient’s spouse talk about the death. The patient’s spouse is experiencing headaches and fatigue. The patient is having trouble sleeping, has no appetite, and gets choked up most of the time. How should the nurse interpret these findings as the basis for a follow-up assessment? Both the patient and the spouse are likely grieving. A nurse is documenting end-of-life care. Which information will the nurse include in the patient’s electronic medical record? (Select all that apply.) Time and date of death Location of body identification tags Time of body transfer and destination Chapter 38 – Stress and Coping In a natural disaster relief facility, the nurse observes that an older-adult male has a recovery plan, while a 25-year-old male is still overwhelmed by the disaster situation. A nurse is planning care for both patients. Which factors will the nurse consider about the different coping reactions? Maturational and situational factors A woman who was sexually assaulted a month ago presents to the emergency department with reports of recurrent nightmares, fear of going to sleep, repeated vivid memories of the sexual assault, and inability to feel much emotion. Which medical problem will the nurse expect to see documented in the chart? Post-traumatic stress disorder The nurse teaches stress-reduction and relaxation training to a health education group of patients after cardiac bypass surgery. Which level of intervention is the nurse using? Tertiary A nurse is teaching guided imagery to a prenatal class. Which technique did the nurse describe? Using sensory peaceful words After a natural disaster occurred, an emergency worker referred a family for crisis intervention services. One family member refused to attend the services, stating, “No way, I’m not crazy.” What is the nurse’s best response? “Seeking this kind of help does not mean that you have a mental illness; it is a short-term problem-solving technique.” A preadolescent patient is experiencing maturational stress. Which area will the nurse focus on when planning care? Self-esteem issues A nurse is caring for a patient with stress and is in the evaluation stage of the critical thinking model. Which actions will the nurse take? Reassess patient’s stress-related symptoms and compare with expected outcomes. An adult male reports a new-onset, seizure-like activity. An EEG and a neurology consultant’s report rule out a seizure disorder. It is determined the patient is using conversion. Which action should the nurse take next? Obtain history of any recent life stressors. A senior college student visits the college health clinic about a freshman student living on the same dormitory floor. The senior student reports that the freshman is crying and is not adjusting to college life. The clinic nurse recognizes this as a combination of situational and maturational stress factors. Which is the best response by the nurse? “Give the freshman student this list of university and community resources.” Despite working in a highly stressful nursing unit and accepting additional shifts, a new nurse has a strategy to prevent burnout. Which strategy will be best for the nurse to use? Strengthen friendships outside the workplace. A female teen with celiac disease continues to eat food she knows will make her ill several hours after ingestion. While planning care, the nurse considers maturational and tertiary-level interventions. Which intervention will the nurse add to the care plan? Assist the teen in meeting dietary restrictions while eating foods similar to those eaten by her friends. A trauma survivor is requesting sleep medication because of “bad dreams.” The nurse is concerned that the patient may be experiencing post-traumatic stress disorder (PTSD). Which question is a priority for the nurse to ask the patient? “Are you reliving your trauma?” A patient in a motor vehicle accident states, “I did not run the red light,” despite very clear evidence on the street surveillance tape. Which defense mechanism is the patient using? Denial A nurse is teaching the staff about the general adaptation syndrome. In which order will the nurse list the stages, beginning with the first stage? Resistance > Exhaustion > Alarm // REA A young male patient is diagnosed with testicular cancer. Which action will the nurse take first? Ask about the patient’s priority needs. A nurse is teaching the staff about a nursing theory that views a person, family, or community developing a normal line of defense. Which theory is the nurse describing? Neuman Systems Model An adult who was in a motor vehicle accident is brought into the emergency department by paramedics, who report the following in- transit vital signs: Oral temperature: 99.0° F Pulse: 102 beats/min Respiratory rate: 26 breaths/min Blood pressure: 140/106 Which hormones should the nurse consider as the most likely causes of the abnormal vital signs? Epinephrine and norepinephrine A nurse is planning care for a patient that uses displacement. Which information should the nurse consider when planning interventions? This protects against feelings of worthlessness and anxiety. Which sociocultural finding in the history of a patient will alert the nurse to a possible developmental problem? Prolonged poverty A nurse is helping parents who have a child with attention-deficit/hyperactivity disorder. Which strategy will the nurse share with the parents to reduce stress regarding homework assignments? Time-management skills A nurse is assessing a patient with prolonged stress. Which conditions will the nurse monitor for in this patient? (Select all that apply.) Cancer Diabetes Infections 39 – Activity and Exercise A nurse observes a patient rising from a chair slowly by pushing on the chair arms. Which type of tension and contraction did the nurse observe? Eccentric tension and isotonic contraction A nurse notices that a patient has a structural curvature of the spine associated with vertebral rotation. Which condition will the nurse most likely find documented in the patient’s medical record? Scoliosis A nurse is caring for a patient who has some immobility from noninflammatory joint degeneration. The nurse is teaching the patient about this process. Which information will the nurse include in the teaching session? This involves overgrowth of bone at the articular ends. The nurse is providing care to a patient who is bedridden. The nurse raises the height of the bed. What is the rationale for the nurse’s action? Prevents a shift in the nurse’s base of support. A nurse is following the no-lift policy when working to prevent personal injury. Which type of personal back injury is the nurse most likely trying to prevent? Lumbar The nurse is caring for a patient in the emergency department with an injured shoulder. Which type of joint will the nurse assess? Synovial The nurse is caring for a patient with inner ear problems. Which goal is the priority? Maintain balance. A nurse is teaching a health promotion class about isotonic exercises. Which types of exercises will the nurse give as examples? Swimming, jogging, and bicycling An adolescent tells the nurse that a health professional said the fibrous tissue that connects bone and cartilage was strained in a sporting accident. On which structure will the nurse focus an assessment? Ligament A nurse is developing an exercise plan for a middle-aged patient. In which order will the nurse instruct the patient to execute the plan, beginning with the first step? Assess fitness level. // Assess > Design > Assemble > Get > Monitor // AssessDAGM Design the fitness program. Assemble equipment. Get started The nurse gives instructions to a nursing assistive personnel (NAP) regarding exercise for a patient. Which action by the NAP indicates a correct understanding of the directions? Reports the patient got dizzy after exercising The nurse is starting an exercise program in a local community as a health promotion project. Which information will the nurse include in the teaching session? A cool-down period lasts about 5 to 10 minutes. The patient is eager to begin an exercise program with a 2-mile jog. The nurse instructs the patient to warm up. The patient does not want to waste time with a “warm-up.” Which information will the nurse share with the patient? The warm-up prepares the body and decreases the potential for injury. The nurse is caring for a patient who cannot bear weight but needs to be transferred from the bed to a chair. The nurse decides to use a transportable hydraulic lift. What will the nurse do? Hook longer straps to the bottom of the sling The nurse is preparing to move a patient to a wheelchair. Which action indicates the nurse is following recommendations for safe patient handling? Mentally reviews the transfer steps before beginning A nurse is working in a facility that follows a comprehensive safe patient-handling program. Which finding will alert the nurse to intervene? Mechanical lifts are in a locked closet. The patient is brought to the emergency department with possible injury to the left shoulder. Which area will the nurse assess to best determine joint mobility? The patient’s range of motion The nurse is evaluating care of a patient for crutches. Which finding indicates a successful outcome? The tip of the crutch is 4 to 6 inches anterior to the front of the patient’s shoes. The patient reports being tired and weak and lacks energy. Upon assessment, the nurse finds that patient has gained weight, and blood pressure and pulse are elevated after climbing stairs. Which nursing diagnosis will the nurse add to the care plan? Activity intolerance The patient weighs 450 lbs (204.5 kg) and reports shortness of breath with any exertion. The health care provider has recommended beginning an exercise program. The patient states that she can hardly get out of bed and just cannot do anything around the house. Which nursing diagnosis will the nurse add to the care plan? Activity intolerance related to excessive weight A patient with diabetes mellitus is starting an exercise program. Which types of exercises will the nurse suggest? Low to moderate intensity A patient is admitted with a stroke. The outcome of this disorder is uncertain, but the patient is unable to move the right arm and leg. The nurse starts passive range-of-motion (ROM) exercises. Which finding indicates successful goal achievement? Joint mobility maintained. A nurse is assessing a patient with activity intolerance for possible orthostatic hypotension. Which finding will help confirm orthostatic hypotension? Blood pressure sitting 130/60; blood pressure 110/60 standing The nurse is teaching a patient how to use a cane. Which information will the nurse include in the teaching session? Place the cane on the stronger side of the body. A nurse is assisting the patient to perform isometric exercises. Which action will the nurse take? Stop the exercise if pain is experienced. The nurse is developing a plan of care for a patient diagnosed with activity intolerance. Which strategy will the nurse use to provide the best chance of maintaining patient compliance? Instructing the patient to use an exercise log to record day, time, duration, and responses to exercise activity The nurse is preparing to transfer an uncooperative patient who does not have upper body strength. Which piece of equipment will be best for the nurses to obtain? Full body sling The nurse is teaching a patient how to sit with crutches. In which order will the nurse present the instructions starting with the first step? Place both crutches in one hand > // Place > Transfer > Grasp > Completely // PTGC Transfer weight to crutches and unaffected leg > Grasp arm of chair with free hand > Completely lower self into chair The nurse is caring for a group of patients. Which patient will the nurse see first? A patient with a heart attack doing isometric exercises A nurse is preparing to move a patient who is able to assist. Which principles will the nurse consider when planning for safe patient handling? (Select all that apply.) Face the direction of the movement. Use proper body mechanics. Use arms and legs. A nurse is assessing activity tolerance of a patient. Which areas will the nurse assess? (Select all that apply.) Skeletal abnormalities Emotional factors Pregnancy status Age A nurse is working in a facility that uses no-lift policies. Which benefits will the nurse observe in the facility? (Select all that apply.) Reduced number of work-related injuries Improved health of nurses A nurse writes the following outcomes for a patient who has chronic obstructive pulmonary disease to improve activity level: Diastolic blood pressure will remain below 70 mm Hg with systolic below 130 mm Hg. Resting heart rate will range between 65 and 75. The last goal is that the patient will exercise 3 times a week. Which evaluative findings indicate successful goal achievement? (Select all that apply.) Resting heart rate 70 Blood pressure 126/64 Reports doing resistive training 1 time and aerobics 2 times this week A 55-year-old patient is preparing to start an exercise program. The health care provider wants 60% of maximum target heart rate. Calculate the heart rate that the nurse will add to the care plan as the target heart rate. Record answer as a whole number. maximum heart rate 99 Chapter 40 — Hygiene A nurse is preparing to provide hygiene care. Which principle should the nurse consider when planning hygiene care? No two individuals perform hygiene in the same manner. A patient’s hygiene schedule of bathing and brushing teeth is largely influenced by family customs. For which age group is the nurse most likely providing care? Preschooler The patient has been diagnosed with diabetes. When admitted, the patient is unkempt and is in need of a bath and foot care. When questioned about hygiene habits, the nurse learns the patient takes a bath once a week and a sponge bath every other day. To provide ultimate care for this patient, which principle should the nurse keep in mind? The patient’s illness may require teaching of new hygiene practices. The nurse is caring for a patient who refuses to bathe in the morning. When asked why, the patient says “I always bathe in the evening.” Which action by the nurse is best? Defer the bath until evening and pass on the information to the next shift. A nurse is completing an assessment of the patient. Which principle is a priority? Critical thinking will always be important. When providing hygiene for an older-adult patient, the nurse closely assesses the skin. What is the rationale for the nurse’s action? Less frequent bathing may be required. The nurse is bathing a patient and notices movement in the patient’s hair. Which action will the nurse take? Use gloves to inspect the hair. The patient has been brought to the emergency department following a motor vehicle accident. The patient is unresponsive. The driver’s license states that glasses are needed to operate a motor vehicle, but no glasses were brought in with the patient. Which action should the nurse take next? Stand to the side of the patient’s eye and observe the cornea. A nurse is assessing a patient’s skin. Which patient is most at risk for impaired skin integrity? A patient who is diaphoretic The nurse is caring for a patient who is immobile. The nurse frequently checks the patient for impaired skin integrity. What is the rationale for the nurse’s action? Pressure reduces circulation to affected tissue. The nurse is caring for a patient who has diabetes mellitus and circulatory insufficiency, with peripheral neuropathy and urinary incontinence. On which areas does the nurse focus care? Decreased pain sensation and increased risk of skin impairment The nurse is caring for a patient who has undergone surgery for a broken leg and has a cast in place. What should the nurse do to prevent skin impairment? Assess surfaces exposed to the edges of the cast for pressure areas. Which action by the nurse will be the most important for preventing skin impairment in a mobile patient with local nerve damage? Assess for pain during a bath. After performing foot care, the nurse checks the medical record and discovers that the patient has a foot disorder caused by a virus. Which condition did the nurse most likely observe? Plantar warts The nurse is caring for a patient who is reporting severe foot pain due to corns. The patient has been using oval corn pads to self-treat the corns, but they seem to be getting worse. Which information will the nurse share with the patient? Depending on severity, surgery may be needed to remove the corns. The patient is diagnosed with athlete’s foot (tinea pedis). The patient says that he is relieved because it is only athlete’s foot, and it can be treated easily. Which information should the nurse consider when formulating a response to the patient? Contagious with frequent recurrences When assessing a patient’s feet, the nurse notices that the toenails are thick and separated from the nail bed. What does the nurse most likely suspect is the cause of this condition? Fungi The nurse is providing education about the importance of proper foot care to a patient who has diabetes mellitus. Which primary goal is the nurse trying to achieve? Prevention of amputation The nurse is providing oral care to an unconscious patient and notes that the patient has extremely bad breath. Which term will the nurse use when reporting to the oncoming shift? Halitosis The nurse is caring for a patient with diabetes. Which task will the nurse assign to the nursing assistive personnel? Making an occupied bed The patient is being treated for cancer with weekly radiation therapy to the head and chemotherapy treatments. Which assessment is the priority? Oral cavity The nurse is providing oral care to an unconscious patient. Which action should the nurse take? Suction the oral cavity. The nurse is teaching the patient about flossing and oral hygiene. Which instruction will the nurse include in the teaching session? Flossing removes plaque and tartar from the teeth. The nurse is teaching the parents of a child who has head lice (pediculosis capitis). Which information will the nurse include in the teaching session? Head lice may spread to furniture and other people. A patient has scaling of the scalp. Which term will the nurse use to report this finding to the oncoming staff? Dandruff A nurse is providing a bath. In which order will the nurse clean the body, beginning with the first area? Eyes Eyes > Face > Arms > Hands > Abdomen > Perineum > Back Face Arm and chest Hands and nails Abdomen and legs Perineum Back and buttocks The nurse is caring for a patient who has multiple ticks on lower legs and body. What should the nurse do to rid the patient of ticks? Use blunt tweezers and pull upward with steady pressure. The nurse is providing oral care to a patient. In which order will the nurse clean the oral cavity, starting with the first area? Chewing and inner tooth surfaces Chewing > Outer > Roof > Tongue // CORT Outer tooth surfaces Roof of mouth, gums, and inside cheek Tongue The nurse is caring for an older-adult patient with Alzheimer’s disease who is ambulatory but requires total assistance with activities of daily living (ADLs). The nurse notices that the patient is edentulous. Which area should the nurse assess? Assess oral cavity. A self-sufficient bedridden patient is unable to reach all body parts. Which type of bath will the nurse assign to the nursing assistive personnel? Partial bed bath The nurse is preparing to provide a complete bed bath to an unconscious patient. The nurse decides to use a bag bath. In which order will the nurse clean the body, starting with the first area? Neck, shoulders, and chest > Neck > Abdomen > Legs > Back > Both Abdomen and groin/perineum > Legs, feet, and web spaces > Back of neck, back, and then buttocks > Both arms, both hands, web spaces, and axilla The female nurse is caring for a male patient who is uncircumcised but not ambulatory and has full function of all extremities. The nurse is providing the patient with a partial bed bath. How should perineal care be performed for this patient? Should be done by the patient A nursing assistive personnel (NAP) is providing AM care to patients. Which action by the NAP will require the nurse to intervene? Turning patient’s head with neck injury to side when giving oral care A nurse is providing AM care to patients. Which action will the nurse take? Applies CHG solution to wash perineum of patient with a stroke The nurse is providing a complete bed bath to a patient using a commercial bath cleansing pack (bag bath). What should the nurse do? Allow the skin to air-dry. A nurse is providing perineal care to a female patient. Which washing technique will the nurse use? From pubic area to rectum The nurse is providing perineal care to an uncircumcised male patient. Which action will the nurse take? Retract the foreskin and return it to its natural position when done. Which instruction will the nurse provide to the nursing assistive personnel when providing foot care for a patient with diabetes? Report sores on the patient’s toes. The debilitated patient is resisting attempts by the nurse to provide oral hygiene. Which action will the nurse take next? Insert an oral airway. A nurse is providing oral care to a patient with stomatitis. Which technique will the nurse use? Avoid commercial mouthwashes. The nurse is teaching a patient about contact lens care. Which instructions will the nurse include in the teaching session? Wash and rinse lens storage case daily. The patient reports to the nurse about a perceived decrease in hearing. When the nurse examines the patient’s ear, a large amount of cerumen buildup at the entrance to the ear canal is observed. Which action will the nurse take next? Apply gentle, downward retraction of the ear canal. The patient is being fitted with a hearing aid. In teaching the patient how to care for the hearing aid, which instructions will the nurse provide? Adjust the volume for a talking distance of 1 yard. The patient is reporting an inability to clear nasal passages. Which action will the nurse take? Use gentle suction to prevent tissue damage. A patient uses an in-the-canal hearing aid. Which assessment is a priority? Cerumen buildup The nurse is caring for a patient with cognitive impairments. Which actions will the nurse take during AM care? (Select all that apply.) Increase the frequency of skin assessment. Reduce triggers in the environment. The nurse is caring for a patient who has peripheral neuropathy. Which clinical manifestations does the nurse expect to find upon assessment? (Select all that apply.) Abnormal gait Foot deformities Muscle wasting of lower extremities A nurse is providing hygiene care to a bariatric patient using chlorhexidine gluconate (CHG) wipes. Which actions will the nurse take? (Select all that apply.) Do not rinse. Clean under breasts. Inform that the skin will feel sticky. Which patients will the nurse determine are in need of perineal care? (Select all that apply.) A patient with rectal and genital surgical dressings A patient with urinary and fecal incontinence A patient who has an indwelling catheter A bariatric patient The patient must stay in bed for a bed change. Which actions will the nurse implement? (Select all that apply.) Advise patient will feel a lump when rolling over. Turn clean pillowcase inside out over the hand holding it. Make a modified mitered corner with sheet, blanket, and spread. Chapter 41—Oxygenation Left-sided heart failure results in pulmonary congestion, the signs and symptoms of which include shortness of breath, cough, crackles, and paroxysmal nocturnal dyspnea (difficulty breathing when lying flat). Right-sided heart failure is systemic and results in peripheral edema, weight gain, and distended neck veins. A nurse is teaching staff about the conduction of the heart. In which order will the nurse present the conduction cycle, starting with the first structure? Sinoatrial (SA) node // SIA BP Intraatrial pathways Atrioventricular (AV) node Bundle of His Purkinje network A nurse is teaching the patient with mitral valve problems about the valves in the heart. Starting on the right side of the heart, describe the sequence of the blood flow through these valves. Tricuspid > Pulmonic > Mitral > Aortic // TPMA A nurse explains the function of the alveoli to a patient with respiratory problems. Which information about the alveoli’s function will the nurse share with the patient? Carries out gas exchange A nurse auscultates heart sounds. When the nurse hears S2, which valves is the nurse hearing close? Aortic and pulmonic The nurse is teaching about the process of exchanging gases through the alveolar capillary membrane. Which term will the nurse use to describe this process? Diffusion A nurse is caring for a patient who was in a motor vehicle accident that resulted in cervical trauma to C4. Which assessment is the priority? Respirations The patient is breathing normally. Which process does the nurse consider is working properly when the patient inspires? Stimulation of chemical receptors in the aorta The home health nurse recommends that a patient with respiratory problems install a carbon monoxide detector in the home. What is the rationale for the nurse’s action? Carbon monoxide tightly binds to hemoglobin, causing hypoxia. While performing an assessment, the nurse hears crackles in the patient’s lung fields. The nurse also learns that the patient is sleeping on three pillows to help with the difficulty breathing during the night. Which condition will the nurse most likely observe written in the patient’s medical record? Left-sided heart failure A patient has a myocardial infarction. On which primary blood vessel will the nurse focus care to reduce ischemia? Coronary artery A nurse is teaching a health class about the heart. Which information from the class members indicates teaching by the nurse is successful for the flow of blood through the heart, starting in the right atrium? Right ventricle, left atrium, left ventricle The nurse suspects the patient has increased afterload. Which piece of equipment should the nurse obtain to determine the presence of this condition? Blood pressure cuff A patient has heart failure and cardiac output is decreased. Which formula can the nurse use to calculate cardiac output? Stroke volume × Heart rate A patient’s heart rate increased from 94 to 164 beats/min. What will the nurse expect? Decrease in cardiac output The nurse is careful to monitor a patient’s cardiac output. Which goal is the nurse trying to achieve? To determine peripheral extremity circulation A nurse is caring for a group of patients. Which patient should the nurse see first? A patient with hypercapnia wearing an oxygen mask A patient has inadequate stroke volume related to decreased preload. Which treatment does the nurse prepare to administer? Intravenous (IV) fluids A nurse is preparing to suction a patient. The pulse is 65 and pulse oximetry is 94%. Which finding will cause the nurse to stop suctioning? Oxygen saturation 88% The patient has right-sided heart failure. Which finding will the nurse expect when performing an assessment? Peripheral edema A nurse is reviewing the electrocardiogram (ECG) results. Which portion of the conduction system does the nurse consider when evaluating the P wave? SA node A nurse teaches a patient about atelectasis. Which statement by the patient indicates an understanding of atelectasis? “It is important to do breathing exercises every hour to prevent atelectasis.” The nurse is caring for a patient with respiratory problems. Which assessment finding indicates a late sign of hypoxia? Cyanosis A nurse is caring for a 5-year-old patient whose temperature is 101.2° F. The nurse expects this patient to hyperventilate. Which factor does the nurse remember when planning care for this type of hyperventilation? Increased metabolic demands A nurse is preparing a patient for nasotracheal suctioning. In which order will the nurse perform the steps, beginning with the first step? Attach catheter to suction system. // Attach > Have > Insert > Apply > Encourage > Rinse Have patient deep breathe. Insert catheter. Apply suction and remove. Encourage patient to cough. Rinse catheter and connecting tubing. A patient has carbon dioxide retention from lung problems. Which type of diet will the nurse most likely suggest for this patient? Low-carbohydrate A nurse is caring for a patient who is taking warfarin (Coumadin) and discovers that the patient is taking garlic to help with hypertension. Which condition will the nurse assess for in this patient? Bleeding A nurse is caring for a patient who has poor tissue perfusion as the result of hypertension. When the patient asks what to eat for breakfast, which meal should the nurse suggest? A cup of nonfat yogurt with granola and a handful of dried apricots Upon auscultation of the patient’s chest, the nurse hears a whooshing sound at the fifth intercostal space. What does this finding indicate to the nurse? Regurgitation of the mitral valve A nurse is caring for a patient with chronic obstructive pulmonary disease (COPD) who is receiving 2 L/min of oxygen. Which oxygen delivery device is most appropriate for the nurse to administer the oxygen? Nasal cannula The nurse needs to closely monitor the oxygen status of an older-adult patient undergoing anesthesia because of which age-related change? Diminished respiratory muscle strength may cause poor chest expansion. The nurse determines that an older-adult patient is at risk for infection due to decreased immunity. Which plan of care best addresses the prevention of infection for the patient? Encourage the patient to stay up-to-date on all vaccinations. A nurse is caring for a patient with continuous cardiac monitoring for heart dysrhythmias. Which rhythm will cause the nurse to intervene immediately? Ventricular tachycardia The patient is experiencing angina pectoris. Which assessment finding does the nurse expect when conducting a history and physical examination? Experiences chest pain after eating a heavy meal A nurse is teaching about risk factors for cardiopulmonary disease. Which risk factor should the nurse describe as modifiable? Stress The nurse is creating a plan of care for an obese patient who is suffering from fatigue related to ineffective breathing. Which intervention best addresses a short-term goal the patient could achieve? Sleeping on two to three pillows at night A nurse is caring for a patient with left-sided hemiparesis who has developed bronchitis and has a heart rate of 105 beats/min, blood pressure of 156/90 mm Hg, and respiration rate of 30 breaths/min. Which nursing diagnosis is a priority? Impaired gas exchange Which nursing intervention is most effective in preventing hospital-acquired pneumonia in an older-adult patient? Assist the patient to cough, turn, and deep breathe every 2 hours. The nurse is assessing a patient with emphysema. Which assessment finding requires further follow-up with the health care provider? Hemoptysis A patient with chronic obstructive pulmonary disease (COPD) asks the nurse why clubbing occurs. Which response by the nurse is most therapeutic? “Your disease doesn’t send enough oxygen to your fingers.” A patient with a pneumothorax has a chest tube inserted and is placed on low constant suction. Which finding requires immediate action by the nurse? No bubbling is present in the suction control chamber of the drainage device. The nurse is caring for a patient with a tracheostomy tube. Which nursing intervention is most effective in promoting effective airway clearance? Administering humidified oxygen through a tracheostomy collar The nurse is educating a student nurse on caring for a patient with a chest tube. Which statement from the student nurse indicates successful learning? “I should report if I see continuous bubbling in the water-seal chamber.” Which coughing technique will the nurse use to help a patient clear central airways? Huff The nurse is suctioning a patient with a tracheostomy tube. Which action will the nurse take? Limit the length of suctioning to 10 seconds. The nurse is caring for a patient who needs oxygen via a nasal cannula. Which task can the nurse delegate to the nursing assistive personnel? Applying the nasal cannula The nurse is using a closed suction device. Which patient will be most appropriate for this suctioning method? A 24-year-old with acute respiratory distress syndrome requiring mechanical ventilation While the nurse is changing the ties on a tracheostomy collar, the patient coughs, dislodging the tracheostomy tube. Which action will the nurse take first? Insert a spare tracheostomy with the obturator. A nurse is following the Ventilator Bundle standards to prevent ventilator-associated pneumonia. Which strategies is the nurse using? (Select all that apply.) Daily oral care with chlorhexidine Cuff monitoring for adequate seal Daily “sedation vacations” A nurse is teaching a community health promotion class and discusses the flu vaccine. Which information will the nurse include in the teaching session? (Select all that apply.) It is given yearly. The inactivated flu vaccine is given to people over 50. A nurse is caring for a patient with sleep apnea. Which types of ventilator support should the nurse be prepared to administer for this patient? (Select all that apply.) Bilevel positive airway pressure (BiPAP) Continuous positive airway pressure (CPAP) Chapter 42—Fluid, Electrolyte, and Acid-Base Balance A patient has dehydration. While planning care, the nurse considers that the majority of the patient’s total water volume exists in with compartment? Intracellular The nurse is teaching about the process of passively moving water from an area of lower particle concentration to an area of higher particle concentration. Which process is the nurse describing? Osmosis The nurse observes edema in a patient who has venous congestion from right heart failure. Which type of pressure facilitated the formation of the patient’s edema? Hydrostatic The nurse administers an intravenous (IV) hypertonic solution to a patient. In which direction will the fluid shift? From intracellular to extracellular A nurse is preparing to start peripheral intravenous (IV) therapy. In which order will the nurse perform the steps starting with the first one? Apply tourniquet. Apply > Select > Release > Clean > Reapply > Insert > Advance Select vein. Release tourniquet. Clean site. Reapply tourniquet. Insert vascular access device. Advance and secure. The nurse is reviewing laboratory results. Which cation will the nurse observe is the mostabundant in the blood? Sodium The nurse receives the patient’s most recent blood work results. Which laboratory value is of greatest concern? Calcium of 15.5 mg/dL The nurse observes that the patient’s calcium is elevated. When checking the phosphate level, what does the nurse expect to see? Decreased Four patients arrive at the emergency department at the same time. Which patient will the nurse see first? An infant with temperature of 102.2° F and diarrhea for 3 days A 2-year-old child is brought into the emergency department after ingesting a medication that causes respiratory depression. For which acid-base imbalance will the nurse most closely monitor this child? Respiratory acidosis A patient is admitted for a bowel obstruction and has had a nasogastric tube set to low intermittent suction for the past 3 days. Which arterial blood gas values will the nurse expect to observe? Metabolic alkalosis The patient has an intravenous (IV) line and the nurse needs to remove the gown. In which order will the nurse perform the steps, starting with the first one? Apply tourniquet. Apply > Select > Release > Clean > Reapply > Insert > Advance Select vein. Release tourniquet. Clean site. Reapply tourniquet. Insert vascular access device. Advance and secure. Which blood gas result will the nurse expect to observe in a patient with respiratory alkalosis? pH 7.53, PaCO2 30 mm Hg, HCO3– 24 mEq/L A nurse is caring for a patient whose ECG presents with changes characteristic of hypokalemia. Which assessment finding will the nurse expect? Abdominal distention In which patient will the nurse expect to see a positive Chvostek sign? A 24-year-old adult admitted for chronic alcohol abuse A patient is experiencing respiratory acidosis. Which organ system is responsible for compensation in this patient? Renal A nurse is caring for a patient with peripheral intravenous (IV) therapy. Which task will the nurse assign to the nursing assistive personnel? Recording intake and output The nurse is caring for a diabetic patient in renal failure who is in metabolic acidosis. Which laboratory findings are consistent with metabolic acidosis? pH 7.3, PaCO2 36 mm Hg, HCO3– 19 mEq/L The nurse is assessing a patient and finds crackles in the lung bases and neck vein distention. Which action will the nurse take first? Raise head of bed. A chemotherapy patient has gained 5 pounds in 2 days. Which assessment question by the nurse is most appropriate? “How many times a day do you urinate?” The health care provider has ordered a hypotonic intravenous (IV) solution to be administered. Which IV bag will the nurse prepare? 0.45% sodium chloride (1/2 NS) The health care provider asks the nurse to monitor the fluid volume status of a heart failure patient and a patient at risk for clinical dehydration. Which is the most effective nursing intervention for monitoring both of these patients? Weigh the patients every morning before breakfast. A nurse is caring for a cancer patient who presents with anorexia, blood pressure 100/60, and elevated white blood cell count. Which primary purpose for starting total parenteral nutrition (TPN) will the nurse add to the care plan? Replace fluid, electrolytes, and nutrients. A patient presents to the emergency department with reports of vomiting and diarrhea for the past 48 hours. The health care provider orders isotonic intravenous (IV) therapy. Which IV will the nurse prepare? 0.9% sodium chloride (NS) A nurse is administering a diuretic to a patient and teaching the patient about foods to increase. Which food choices by the patient will best indicate successful teaching? Potatoes and fresh fruit The nurse is evaluating the effectiveness of the intravenous fluid therapy in a patient with hypernatremia. Which finding indicates goal achievement? Serum sodium concentration returns to normal. The nurse is calculating intake and output on a patient. The patient drinks 150 mL of orange juice at breakfast, voids 125 mL after breakfast, vomits 250 mL of greenish fluid, sucks on 60 mL of ice chips, and for lunch consumes 75 mL of chicken broth. Which totals for intake and output will the nurse document in the patient’s medical record? Intake 255; output 375 A nurse is assessing a patient. Which assessment finding should cause a nurse to further assess for extracellular fluid volume deficit? Postural hypotension A patient is to receive 1000 mL of 0.9% sodium chloride intravenously at a rate of 125 mL/hr. The nurse is using microdrip gravity drip tubing. Which rate will the nurse calculate for the minute flow rate (drops/min)? 125 drops/min A nurse begins infusing a 250-mL bag of IV fluid at 1845 on Monday and programs the pump to infuse at 50 mL/hr. At what time should the infusion be completed? 23:45 Monday A nurse is caring for a diabetic patient with a bowel obstruction and has orders to ensure that the volume of intake matches the output. In the past 4 hours, the patient received dextrose 5% with 0.9% sodium chloride through a 22-gauge catheter infusing at 150 mL/hr and has eaten 200 mL of ice chips. The patient also has an NG suction tube set to low continuous suction that had 300-mL output. The patient has voided 400 mL of urine. After reporting these values to the health care provider, which order does the nurse anticipate? Add a potassium supplement to replace loss from output. A nurse is caring for a patient who is receiving peripheral intravenous (IV) therapy. When the nurse is flushing the patient’s peripheral IV, the patient reports pain. Upon assessment, the nurse notices a red streak that is warm to the touch. What is the nurse’s initial action? Discontinue the IV. A nurse is assisting the health care provider in inserting a central line. Which action indicates the nurse is following the recommended bundle protocol to reduce central line-associated bloodstream infections (CLABSI)? Uses chlorhexidine skin antisepsis prior to insertion. The nurse is caring for a group of patients. Which patient will the nurse see first? A patient with D5W hanging with the blood A nurse is administering a blood transfusion. Which assessment finding will the nurse report immediately? Temperature 101.3° F A nurse has just received a bag of packed red blood cells (RBCs) for a patient. What is the longest time the nurse can let the blood infuse? 4 hours A patient has an acute intravascular hemolytic reaction to a blood transfusion. After discontinuing the blood transfusion, which is the nurse’s next action? Start normal saline at TKO rate using new tubing. A nurse is assessing a patient who is receiving a blood transfusion and finds that the patient is anxiously fidgeting in bed. The patient is afebrile and dyspneic. The nurse auscultates crackles in both lung bases and sees jugular vein distention. On which transfusion complication will the nurse focus interventions? Fluid volume overload A nurse is preparing to start a blood transfusion. Which type of tubing will the nurse obtain? A filter to ensure that clots do not enter the patient The nurse is caring for a patient with hyperkalemia. Which body system assessment is the priority? Cardiac Which assessment finding will the nurse expect for a patient with the following laboratory values: sodium 145 mEq/L, potassium 4.5 mEq/L, calcium 4.5 mg/dL? Tingling of extremities with possible tetany While the nurse is taking a patient history, the nurse discovers the patient has a type of diabetes that results from a head injury and does not require insulin. Which dietary change should the nurse share with the patient? Drink plenty of fluids throughout the day to stay hydrated. A nurse is selecting a site to insert an intravenous (IV) catheter on an adult. Which actions will the nurse take? (Select all that apply.) Check for contraindications to the extremity. Choose a vein with minimal curvature. Avoid areas of flexion. Which assessments will alert the nurse that a patient’s IV has infiltrated? (Select all that apply.) Edema of the extremity near the insertion site Skin discolored or pale in appearance Skin cool to the touch A nurse is discontinuing a patient’s peripheral IV access. Which actions should the nurse take? (Select all that apply.) Stop the infusion before removing the IV catheter. Keep the catheter parallel to the skin while removing it. Apply pressure to the site for 2 to 3 minutes after removal. A patient has 250 mL of a jejunostomy feeding with 30 mL of water before and after feeding and 200 mL of urine. Thirty minutes later the patient has 100 mL of diarrhea. At 1300 the patient receives 150 mL of blood and voids another 200 mL. Calculate the patient’s intake. Record your answer as a whole number. mL 460 A nurse is monitoring patients for fluid and electrolyte and acid-base imbalances. Match the body’s regulators to the function it provides. Increases excretion of sodium and water Atrial natriuretic peptide Reduces excretion of sodium and water Aldosterone Reduces excretion of water Antidiuretic hormone Major buffer in the extracellular fluid Bicarbonate Vasoconstricts and stimulates aldosterone release Angiotensin II Chapter 43—Sleep The nurse is caring for a young-adult patient on the medical-surgical unit. When doing midnight checks, the nurse observes the patient awake, putting a puzzle together. Which information will the nurse consider to best explain this finding? The patient’s sleep-wake cycle preference is late evening. The nurse is providing an educational session on sleep regulation for new nurses in the Sleep Disorder Treatment Center. Which statement by the nurses will best indicate that the teaching is effective? “If the patient has a disease process in the central nervous system, it can influence the functions of sleep.” The nurse is caring for a patient who is having trouble sleeping. Which action will the nurse take? Encourage deep breathing. The nurse is caring for a patient in the sleep lab. Which assessment finding indicates to the nurse that the patient is in stage 4 NREM? The patient is difficult to awaken. A nurse is teaching the staff about the sleep cycle. Which sequence will the nurse include in the teaching session? NREM Stage 1, 2, 3, 4, 3, 2, REM Which nursing observation of the patient in intensive care indicates the patient is sleeping comfortably during NREM sleep? Eyes closed, lying quietly, respirations 12, heart rate 60 The nurse is teaching a new mother about the sleep requirements of a neonate. Which comment by the patient indicates a correct understanding of the teaching? “I will ask my mom to come after the first week, when the baby is more alert.” The nurse is discussing lack of sleep with a middle-aged adult. Which area should the nurse most likely assess to determine a possible cause of the lack of sleep? Anxiety A single parent is discussing the sleep needs of a preschooler with the nurse. Which information will the nurse share with the parent? “Preschoolers may have trouble settling down after a busy day.” The nurse is having a conversation with an adolescent regarding the need for sleep. The adolescent states that it is common to stay up with friends several nights a week. Which action should the nurse take next? Discuss with the adolescent sleep needs and the effects of excessive daytime sleepiness. The nurse is completing an assessment on an older-adult patient who is having difficulty falling asleep. Which condition will the nurse further assess for in this patient? Depression The nurse is caring for an adolescent with an appendectomy who is reporting difficulty falling asleep. Which intervention will be most appropriate? Encourage the discontinuation of a soda and chocolate nightly snack. A patient has obstructive sleep apnea. Which assessment is the priority? Respiratory status The patient has just been diagnosed with narcolepsy. The nurse teaches the patient about management of the condition. Which information from the patient will cause the nurse to intervene? Sits in hot, stuffy rooms The nurse is caring for a patient who has been in holding in the emergency department for 24 hours. The nurse is concerned about the patient’s experiencing sleep deprivation. Which action will be best for the nurse to take? Expedite the process of obtaining a medical-surgical room for the patient. The nurse is completing a sleep assessment on a patient. Which tool will the nurse use to complete the assessment? Visual analog scale The nurse is beginning a sleep assessment on a patient. Which question will be most appropriate for the nurse to ask initially? “How are you sleeping?” The nurse adds a nursing diagnosis of ineffective breathing pattern to a patient’s care plan. Which sleep condition caused the nurse to assign this nursing diagnosis? Obstructive sleep apnea The nurse is caring for a postpartum patient. The patient’s labor has lasted over 28 hours within the hospital; the patient has not slept and is disoriented to date and time. Which nursing diagnosis will the nurse document in the patient’s care plan? Sleep deprivation The patient presents to the clinic with reports of irritability, being sleepy during the day, chronically not being able to fall asleep, and being tired. Which nursing diagnosis will the nurse document in the plan of care? Insomnia The nurse is preparing an older-adult patient’s evening medications. Which treatment will the nurse recognize as relatively safe for difficulty sleeping in older adults? Ramelteon (Rozerem) The nurse is caring for a patient on the medical-surgical unit who is experiencing an exacerbation of asthma. Which intervention will be most appropriate to help this patient sleep? Place bed in semi-Fowler’s position. A young mother has been hospitalized for an irregular heartbeat (dysrhythmia). The night nurse makes rounds and finds the patient awake. Which action by the nurse is mostappropriate? Take time to sit and talk with the patient about her inability to sleep. The nurse is evaluating outcomes for the patient with insomnia. Which key principle will the nurse consider during this process? The patient is the best evaluator of sleep. A patient has sleep deprivation. Which statement by the patient will indicate to the nurse that outcomes are being met? “I feel rested when I wake up in the morning.” An older-adult patient is visiting the clinic after a fall during the night. The nurse obtains information on what medications the patient takes. Which medication most likely contributed to the patient’s fall? Benzodiazepine The nurse is caring for a patient who has not been able to sleep well while in the hospital, leading to a disrupted sleep-wake cycle. Which assessment findings will the nurse monitor for in this patient? (Select all that apply.) Changes in physiological function such as temperature Decreased appetite and weight loss Anxiety, irritability, and restlessness Impaired judgment The nurse is caring for a patient in the intensive care unit who is having trouble sleeping. The nurse explains the purpose of sleep and its benefits. Which information will the nurse include in the teaching session? (Select all that apply.) NREM sleep contributes to body tissue restoration. Restful sleep preserves cardiac function. Sleep contributes to cognitive restoration. The patient and the nurse discuss the need for sleep. After the discussion, the patient is able to state factors that hinder sleep. Which statements indicate the patient has a good understanding of the teaching? (Select all that apply.) “Drinking coffee at 7 PM could interrupt my sleep.” “Staying up late for a party can interrupt sleep patterns.” “Changing the time of day that I eat dinner can disrupt sleep.” “Worrying about work can disrupt my sleep.” A community health nurse is providing an educational session at the senior center on how to promote sleep. Which practices should the nurse recommend? (Select all that apply.) Sleep where you sleep best. Use sedatives as a last resort. Decrease fluids 2 to 4 hours before sleep. Get up if unable to fall asleep in 15 to 30 minutes. Chapter 44—Pain Management An oriented patient has recently had surgery. Which action is best for the nurse to take to assess this patient’s pain? Ask the patient to rate the level of pain. A nurse is caring for a patient who recently had abdominal surgery and is experiencing severe pain. The patient’s blood pressure is 110/60 mm Hg, and heart rate is 60 beats/min. Additionally, the patient does not appear to be in any distress. Which response by the nurse is mosttherapeutic? “What would you like to try to alleviate your pain?” A nurse teaches the patient about the gate control theory. Which statement made by a patient reflects a correct understanding about the relationship between the gate control theory of pain and the use of meditation to relieve pain? “Meditation controls pain by blocking pain impulses from coming through the gate.” A nurse is planning care for an older-adult patient who is experiencing pain. Which statement made by the nurse indicates the supervising nurse needs to follow up? “As adults age, their ability to perceive pain decreases.” The nurse is caring for two patients; both are having a hysterectomy. The first patient is having the hysterectomy after a complicated birth. The second patient has uterine cancer. What will most likely influence the experience of pain for these two patients? Meaning of pain The nurse is preparing pain medications. To which patient does the nurse anticipate administering an opioid fentanyl patch? A 50-year-old patient with prostate cancer A patient is receiving opioid medication through an epidural infusion. Which action will the nurse take? Label the tubing that leads to the epidural catheter. A woman is in labor and refuses to receive any sort of anesthesia medication. Which alternative treatment is best for this patient? Relaxation and guided imagery A nurse is teaching a patient about patient-controlled analgesia (PCA). Which statement made by the patient indicates to the nurse that teaching is effective? “I feel less anxiety about the possibility of overdosing.” A nurse is caring for a patient who is experiencing pain following abdominal surgery. Which information is important for the nurse to share with the patient when providing patient education about effective pain management? “We should work together to create a schedule to provide regular dosing of medication.” A nurse is caring for a patient who recently had spinal surgery. The nurse knows that patients usually experience acute pain following this type of surgery. The patient refuses to get up and walk and is not moving around in the bed. However, the patient is stoic and denies experiencing pain at this time. What most likely explains this patient’s behavior? The patient’s culture is possibly influencing the patient’s experience of pain. A nurse is providing discharge teaching for a patient with a fractured humerus. The patient is going home with hydrocodone. Which important patient education does the nurse provide? “You need to drink plenty of fluids and eat a diet high in fiber.” A patient arrives at the emergency department experiencing a headache and rates the pain as 7 on a 0 to 10 pain scale. Which nonpharmacological intervention does the nurse implement for this patient while awaiting orders for pain medication from the health care provider? Softly plays music that the patient finds relaxing A patient who has had type 2 diabetes for 26 years is beginning to experience peripheral neuropathy in the feet and lower leg. The nurse is providing education to the patient to prevent injury to the feet by wearing shoes or slippers when walking. Which statement made by the nurse best explains the rationale for this instruction? “If you step on something without shoes, you might not feel it; this could possibly cause injury to your foot.” A nurse is assessing a patient who started to have severe pain 3 days ago. When the nurse asks the patient to describe the pain, the patient states, “The pain feels like it is in my stomach. It is a burning pain, and it spreads out in a circle around the spot where it hurts the most.” Which type of pain does the nurse document the patient is having at this time? Visceral pain A patient who had a motor vehicle crash 2 days ago is experiencing pain and is receiving patient-controlled analgesia (PCA). Which assessment finding indicates effective pain management with the PCA? The patient rates pain at a level of 2 on a 0 to 10 scale. The nurse is caring for a patient to ease modifiable factors that contribute to pain. Which areas did the nurse focus on with this patient? Anxiety and fear The nurse is evaluating the effectiveness of guided imagery for pain management as used for a patient who has second- and third- degree burns and needs extensive dressing changes. Which finding best indicates the effectiveness of guided imagery? The patient’s need for analgesic medication decreases during the dressing changes. A nurse is providing medication education to a patient who just started taking ibuprofen. Which information will the nurse include in the teaching session? Ibuprofen inhibits the production of prostaglandins. The nurse has brought a patient the scheduled pain medication. The patient asks the nurse to wait to give pain medication until the time for the dressing change, which is 2 hours away. Which response by the nurse is most therapeutic? “Would you like medication to be given for dressing changes in addition to your regularly scheduled medication?” A nurse receives an order from a health care provider to administer hydrocodone and acetaminophen (Vicodin ES 7.5/750), to a patient who is experiencing 8/10 postsurgical pain. The order is to give 2 tablets every 6 hours by mouth as needed for pain. What is the nurse’s next best action? Ask the health care provider to verify the dosage and frequency of the medication. The nurse is caring for a 4-year-old child who has pain. Which technique will the nurse use tobest assess pain in this child? Use the FACES scale. A nurse is caring for a group of patients. Which patient will the nurse see first? A patient who received morphine and has a pulse of 62 beats/min, respirations 10 breaths/min, and blood pressure 110/60 mm Hg A nurse is caring for a patient with chronic pain. Which statement by the nurse indicates an understanding of pain management? “I need to reassess the patient’s pain 1 hour after administering oral pain medication.” The nurse is assessing how a patient’s pain is affecting mobility. Which assessment question is most appropriate? “What activities, if any, has your pain prevented you from doing?” The nurse is teaching a student nurse about pain assessment scales. Which statement by the student indicates effective teaching? “You cannot use a pain scale to compare the pain of my patient with the pain of your patient.” The nurse is administering pain medication for several patients. Which patient does the nurse administer medication to first? The patient who is experiencing 8/10 pain and has an immediate order for pain medication The nurse is assessing a patient for opioid tolerance. Which finding supports the nurse’s assessment? The patient needs increasingly higher doses of opioid to control pain. A nurse is caring for a patient with rheumatoid arthritis who is now going to be taking 2 acetaminophen (Tylenol) tablets every 6 hours to control pain. Which part of the patient’s social history is the nurse most concerned about? Patient drinks 1 to 2 glasses of wine every night. The nurse is caring for a patient who suddenly experiences chest pain. What is the nurse’s first priority? Ask the patient to rate and describe the pain. The nurse is caring for a group of patients. Which task may the nurse delegate to the nursing assistive personnel (NAP)? Administer a back massage to a patient with pain. A nurse is caring for a patient with chronic pain from arthritis. Which action is best for the nurse to take? Give pain medications around the clock. A nurse is caring for a patient who fell on the ice and has connective tissue damage in the wrist and hand. The patient describes the pain as throbbing. Which type of pain does the nurse document in this patient’s medical record? Somatic pain The nurse is caring for an infant in the intensive care unit. Which information should the nurse consider when planning care for this patient? Infants respond behaviorally and physiologically to painful stimuli. The nurse is administering ibuprofen (Advil) to an older patient. Which assessment data causes the nurse to hold the medication? (Select all that apply.) Patient states allergy to aspirin. Patient reports past medical history of gastric ulcer. Chapter 45—Nutrition A nurse is teaching about the energy needed at rest to maintain life-sustaining activities for a specific period of time. What is the nurse discussing? Basal metabolic rate (BMR) In general, when a patient’s energy requirements are completely met by kilocalorie (kcal) intake in food, which assessment finding will the nurse observe? Weight does not change. A nurse is teaching a patient about proteins that must be obtained through the diet and cannot be synthesized in the body. Which term used by the patient indicates teaching is successful? Indispensable amino acids A nurse is caring for a patient with a postsurgical wound. When planning care, which goal will be the priority? Promote positive nitrogen balance. In providing diet education for a patient on a low-fat diet, which information is important for the nurse to share? Saturated fats are found mostly in animal sources. A patient has a decreased gag reflex, left-sided weakness, and drooling. Which action will the nurse take when feeding this patient? Flex head with chin tuck. The patient has been diagnosed with cardiovascular disease and placed on a low-fat diet. The patient asks the nurse, “How much fat should I have? I guess the less fat, the better.” Which information will the nurse include in the teaching session? Deficiencies occur when fat intake falls below 10% of daily nutrition. The nurse is describing the ChooseMyPlate program to a patient. Which statement from the patient indicates successful learning? “I can use this to make healthy lifestyle food choices.” The nurse is teaching a health class about the ChooseMyPlate program. Which guidelines will the nurse include in the teaching session? Balancing calories The nurse is providing nutrition education to a Korean patient using the five food groups. In doing so, what should be the focus of the teaching? Including racial and ethnic practices with food preferences of the patient A nurse is teaching a nutrition class about the different daily values. When teaching about the referenced daily intakes (RDIs), which information should the nurse include? Have values for protein, vitamins, and minerals The nurse is planning care for a group of patients. Which task will the nurse assign to the nursing assistive personnel? Measuring capillary blood glucose level In teaching mothers-to-be about infant nutrition, which instruction should the nurse provide? Provide breast milk or formula for the first 4 to 6 months. When planning care for an adolescent who plays sports, which modification should the nurse include in the care plan? Increasing carbohydrates to 55% to 60% of total intake In providing prenatal care to a pregnant patient, what does the nurse teach the expectant mother? Folic acid is needed to help prevent birth defects and anemia. The patient is an 80-year-old male who is visiting the clinic today for a routine physical examination. The patient’s skin turgor is fair, but the patient reports fatigue and weakness. The skin is warm and dry, pulse rate is 116 beats/min, and urinary sodium level is slightly elevated. Which instruction should the nurse provide? Drink more water to prevent further dehydration. The nurse is assessing a patient for nutritional status. Which action will the nurse take? Combine multiple objective measures with subjective measures. The patient has a calculated body mass index (BMI) of 34. How will the nurse classify this finding? Obese A nurse is caring for patients with dysphagia. Which patient has neurogenic dysphagia? A patient with stroke The patient has H. pylori. Which action should the nurse take? Encourage completion of antibiotic therapy. In determining malnourishment in a patient, which assessment finding is consistent with this disorder? Spoon-shaped nails A nurse is preparing to administer an enteral feeding. In which order will the nurse implement the steps, starting with the first one? Elevate head of bed to at least 30 degrees. > // Elevate > Verify > Check > Flush > Initiate // EVCFI Verify tube placement. > Check for gastric residual volume. > Flush tubing with 30 mL of water. > Initiate feeding. The patient is admitted with facial trauma, including a broken nose, and has a history of esophageal reflux and of aspiration pneumonia. With which tube will the nurse most likely administer the feeding? Jejunostomy tube The nurse is preparing to insert a nasogastric tube in a patient who is semiconscious. To determine the length of the tube needed to be inserted, how should the nurse measure the tube? From the tip of the nose to the earlobe to the xiphoid process Before giving the patient an intermittent gastric tube feeding, what should the nurse do? Have the tube feeding at room temperature. A small-bore feeding tube is placed. Which technique will the nurse use to best verify tube placement? X-ray The nurse is concerned about pulmonary aspiration when providing the patient with an intermittent tube feeding. Which action is the priority? Verify tube placement before feeding. The patient is to receive multiple medications via the nasogastric tube. The nurse is concerned that the tube may become clogged. Which action is best for the nurse to take? Check with the pharmacy for availability of the liquid forms of medications. The patient has just started on enteral feedings, and the patient is reporting abdominal cramping. Which action will the nurse take next? Slow the rate of tube feeding. The patient has just been started on an enteral feeding and has developed diarrhea after being on the feeding for 2 hours. What does the nurse suspect is the most likely cause of the diarrhea? Formula intolerance A patient develops a foodborne disease from Escherichia coli. When taking a health history, which food item will the nurse most likely find the patient ingested? Undercooked ground beef The nurse is caring for a patient receiving total parenteral nutrition (TPN). Which action will the nurse take? Wear a sterile mask when changing the central venous catheter dressing. The patient is having at least 75% of nutritional needs met by enteral feeding, so the health care provider has ordered the parenteral nutrition (PN) to be discontinued. However, the nurse notices that the PN infusion has fallen behind. What should the nurse do? Taper infusion gradually. The patient is on parenteral nutrition and is lethargic. The patient reports thirst and headache and has had increased urination. Which problem does the nurse prepare to address? Hyperglycemia In providing diabetic teaching for a patient with type 1 diabetes mellitus, which instructions will the nurse provide to the patient? Saturated fat should be limited to less than 7% of total calories. The patient with cardiovascular disease is receiving dietary instructions from the nurse. Which information from the patient indicates teaching is successful? Limit trans fat to less than 1%. The nurse is providing home care for a patient diagnosed with acquired immunodeficiency syndrome (AIDS). Which dietary intervention will the nurse add to the care plan? Provide small, frequent nutrient-dense meals for maximizing kilocalories. A patient is on a full liquid diet. Which food item choice by the patient will cause the nurse to intervene? Mashed potatoes and gravy A nurse is caring for a group of patients. Which patient will the nurse see first? Patient receiving total parenteral nutrition infusing with same tubing for 26 hours The nurse is preparing to check the gastric aspirate for pH. Which equipment will the nurse obtain? Asepto syringe A nurse is teaching a health class about the nutritional requirements throughout the life span. Which information should the nurse include in the teaching session? (Select all that apply.) Infants triple weight at 1 year. Toddlers become picky eaters. Older adults have altered food flavor from a decrease in taste cells. The patient is asking the nurse about the best way to stay healthy. The nurse explains to the patient which teaching points? (Select all that apply.) Increase physical activity. Maintain body weight in a healthy range. Choose and prepare foods with little salt. When assessing patient with nutritional needs, which patients will require follow-up from the nurse? (Select all that apply.) A patient with infection taking tetracycline with milk A patient with diverticulitis following a high-fiber diet daily A patient with an enteral feeding and 500 mL of gastric residual To honor cultural values of patients from different ethnic/religious groups, which actions demonstrate culturally sensitive care by the nurse? (Select all that apply.) Allows fasting on Yom Kippur for a Jewish patient Serves no ham products to a Muslim patient Serves no meat or fish to a Hindu patient Chapter 46—Urinary Elimination An oriented patient has recently had surgery. Which action is best for the nurse to take to assess this patient’s pain? Ask the patient to rate the level of pain. A nurse is caring for a patient who recently had abdominal surgery and is experiencing severe pain. The patient’s blood pressure is 110/60 mm Hg, and heart rate is 60 beats/min. Additionally, the patient does not appear to be in any distress. Which response by the nurse is most therapeutic? “What would you like to try to alleviate your pain?” A nurse teaches the patient about the gate control theory. Which statement made by a patient reflects a correct understanding about the relationship between the gate control theory of pain and the use of meditation to relieve pain? “Meditation controls pain by blocking pain impulses from coming through the gate.” A nurse is planning care for an older-adult patient who is experiencing pain. Which statement made by the nurse indicates the supervising nurse needs to follow up? “As adults age, their ability to perceive pain decreases.” The nurse is caring for two patients; both are having a hysterectomy. The first patient is having the hysterectomy after a complicated birth. The second patient has uterine cancer. What will most likely influence the experience of pain for these two patients? Meaning of pain The nurse is preparing pain medications. To which patient does the nurse anticipate administering an opioid fentanyl patch? A 50-year-old patient with prostate cancer A patient is receiving opioid medication through an epidural infusion. Which action will the nurse take? Label the tubing that leads to the epidural catheter. A woman is in labor and refuses to receive any sort of anesthesia medication. Which alternative treatment is best for this patient? Relaxation and guided imagery A nurse is teaching a patient about patient-controlled analgesia (PCA). Which statement made by the patient indicates to the nurse that teaching is effective? “I feel less anxiety about the possibility of overdosing.” A nurse is caring for a patient who is experiencing pain following abdominal surgery. Which information is important for the nurse to share with the patient when providing patient education about effective pain management? “We should work together to create a schedule to provide regular dosing of medication.” A nurse is caring for a patient who recently had spinal surgery. The nurse knows that patients usually experience acute pain following this type of surgery. The patient refuses to get up and walk and is not moving around in the bed. However, the patient is stoic and denies experiencing pain at this time. What most likely explains this patient’s behavior? The patient’s culture is possibly influencing the patient’s experience of pain. A nurse is providing discharge teaching for a patient with a fractured humerus. The patient is going home with hydrocodone. Which important patient education does the nurse provide? “You need to drink plenty of fluids and eat a diet high in fiber.” A patient arrives at the emergency department experiencing a headache and rates the pain as 7 on a 0 to 10 pain scale. Which nonpharmacological intervention does the nurse implement for this patient while awaiting orders for pain medication from the health care provider? Softly plays music that the patient finds relaxing A patient who has had type 2 diabetes for 26 years is beginning to experience peripheral neuropathy in the feet and lower leg. The nurse is providing education to the patient to prevent injury to the feet by wearing shoes or slippers when walking. Which statement made by the nurse best explains the rationale for this instruction? “If you step on something without shoes, you might not feel it; this could possibly cause injury to your foot.” 15.A nurse is assessing a patient who started to have severe pain 3 days ago. When the nurse asks the patient to describe the pain, the patient states, “The pain feels like it is in my stomach. It is a burning pain, and it spreads out in a circle around the spot where it hurts the most.” Which type of pain does the nurse document the patient is having at this time? Visceral pain A patient who had a motor vehicle crash 2 days ago is experiencing pain and is receiving patient-controlled analgesia (PCA). Which assessment finding indicates effective pain management with the PCA? The patient rates pain at a level of 2 on a 0 to 10 scale. The nurse is caring for a patient to ease modifiable factors that contribute to pain. Which areas did the nurse focus on with this patient? Anxiety and fear The nurse is evaluating the effectiveness of guided imagery for pain management as used for a patient who has second- and third- degree burns and needs extensive dressing changes. Which finding best indicates the effectiveness of guided imagery? The patient’s need for analgesic medication decreases during the dressing changes. A nurse is providing medication education to a patient who just started taking ibuprofen. Which information will the nurse include in the teaching session? Ibuprofen inhibits the production of prostaglandins. The nurse has brought a patient the scheduled pain medication. The patient asks the nurse to wait to give pain medication until the time for the dressing change, which is 2 hours away. Which response by the nurse is most therapeutic? “Would you like medication to be given for dressing changes in addition to your regularly scheduled medication?” A nurse receives an order from a health care provider to administer hydrocodone and acetaminophen (Vicodin ES 7.5/750), to a patient who is experiencing 8/10 postsurgical pain. The order is to give 2 tablets every 6 hours by mouth as needed for pain. What is the nurse’s next bestaction? Ask the health care provider to verify the dosage and frequency of the medication. The nurse is caring for a 4-year-old child who has pain. Which technique will the nurse use tobest assess pain in this child? Use the FACES scale. A nurse is caring for a group of patients. Which patient will the nurse see first? A patient who received morphine and has a pulse of 62 beats/min, respirations 10 breaths/min, and blood pressure 110/60 mm Hg A nurse is caring for a patient with chronic pain. Which statement by the nurse indicates an understanding of pain management? I need to reassess the patient’s pain 1 hour after administering oral pain medication.” The nurse is assessing how a patient’s pain is affecting mobility. Which assessment question is most appropriate? “What activities, if any, has your pain prevented you from doing?” The nurse is teaching a student nurse about pain assessment scales. Which statement by the student indicates effective teaching? “You cannot use a pain scale to compare the pain of my patient with the pain of your patient.” The nurse is administering pain medication for several patients. Which patient does the nurse administer medication to first? The patient who is experiencing 8/10 pain and has an immediate order for pain medication The nurse is assessing a patient for opioid tolerance. Which finding supports the nurse’s assessment? The patient needs increasingly higher doses of opioid to control pain. A nurse is caring for a patient with rheumatoid arthritis who is now going to be taking 2 acetaminophen (Tylenol) tablets every 6 hours to control pain. Which part of the patient’s social history is the nurse most concerned about? Patient drinks 1 to 2 glasses of wine every night. The nurse is caring for a patient who suddenly experiences chest pain. What is the nurse’s first priority? Ask the patient to rate and describe the pain. The nurse is caring for a group of patients. Which task may the nurse delegate to the nursing assistive personnel (NAP)? Administer a back massage to a patient with pain. A nurse is caring for a patient with chronic pain from arthritis. Which action is best for the nurse to take? Give pain medications around the clock. A nurse is caring for a patient who fell on the ice and has connective tissue damage in the wrist and hand. The patient describes the pain as throbbing. Which type of pain does the nurse document in this patient’s medical record? Somatic pain The nurse is caring for an infant in the intensive care unit. Which information should the nurse consider when planning care for this patient? Infants respond behaviorally and physiologically to painful stimuli. The nurse is administering ibuprofen (Advil) to an older patient. Which assessment data causes the nurse to hold the medication? (Select all that apply.) Patient states allergy to aspirin. Patient reports past medical history of gastric ulcer. Chapter 47—Bowel Elimination The nurse is teaching a health class about the gastrointestinal tract. The nurse will explain that which portion of the digestive tract absorbs most of the nutrients? Duodenum The nurse is caring for patients with ostomies. In which ostomy location will the nurse expect very liquid stool to be present? Ascending A nurse is teaching a patient about the large intestine in elimination. In which order will the nurse list the structures, starting with the first portion? Cecum, ascending, transverse, descending, sigmoid, and rectum The nurse is planning care for a group of patients. Which task will the nurse assign to the nursing assistive personnel (NAP)? Administering an enema A nurse is assisting a patient in making dietary choices that promote healthy bowel elimination. Which menu option should the nurse recommend? Grape and walnut chicken salad sandwich on whole wheat bread A patient is using laxatives three times daily to lose weight. After stopping laxative use, the patient has difficulty with constipation and wonders if laxatives should be taken again. Which information will the nurse share with the patient? Long-term laxative use causes the bowel to become less responsive to stimuli, and constipation may occur. A patient with a hip fracture is having difficulty defecating into a bedpan while lying in bed. Which action by the nurse will assist the patient in having a successful bowel movement? Raising the head of the bed Which patient is most at risk for increased peristalsis? A 21-year-old female with three final examinations on the same day A patient expresses concerns over having black stool. The fecal occult test is negative. Which response by the nurse is most appropriate? “Do you take iron supplements?” Which patient will the nurse assess most closely for an ileus? A patient with surgery for bowel disease and anesthesia A patient has a fecal impaction. Which portion of the colon will the nurse assess? Rectum The nurse is managing bowel training for a patient. To which patient is the nurse most likely providing care? A 70-year-old patient with stool incontinence Which nursing intervention is most effective in promoting normal defecation for a patient who has muscle weakness in the legs? Use a mobility device to place the patient on a bedside commode. The nurse is devising a plan of care for a patient with the nursing diagnosis of Constipation related to opioid use. Which outcome will the nurse evaluate as successful for the patient to establish normal defecation? The patient reports eliminating a soft, formed stool. The nurse is emptying an ileostomy pouch for a patient. Which assessment finding will the nurse report immediately? Presence of blood in the stool The nurse will anticipate which diagnostic examination for a patient with black tarry stools? Endoscopy The nurse has attempted to administer a tap water enema for a patient with fecal impaction with no success. The fecal mass is too large for the patient to pass voluntarily. Which is the next priority nursing action? Obtaining an order for digital removal of stool A nurse is checking orders. Which order should the nurse question? A Kayexalate enema for a patient with severe hypokalemia The nurse is performing a fecal occult blood test. Which action should the nurse take? Report a positive finding to the provider. A nurse is preparing a patient for a magnetic resonance imaging (MRI) scan. Which nursing action is most important? Removing all of the patient’s metallic jewelry. A patient with a fecal impaction has an order to remove stool digitally. In which order will the nurse perform the steps, starting with the first one? Identify patient using two identifiers. > // Identify > Obtain > Place > Apply > Insert > Massage // IOPAM Obtain baseline vital signs. > Place patient on left side in Sims’ position. > Apply clean gloves and lubricate. > Insert index finger into the rectum. > Massage around the feces and work down to remove. Before administering a cleansing enema to an 80-year-old patient, the patient says “I don’t think I will be able to hold the enema.” Which is the next priority nursing action? Positioning the patient in the dorsal recumbent position on a bedpan A nurse is providing care to a group of patients. Which patient will the nurse see first? An older patient with glaucoma about to receive an enema A patient is diagnosed with a bowel obstruction. Which type of tube is the best for the nurse to obtain for gastric decompression? Salem sump A patient had an ileostomy surgically placed 2 days ago. Which diet will the nurse recommend to the patient to ease the transition of the new ostomy? Turkey meatloaf with white rice and apple juice A nurse is pouching an ostomy on a patient with an ileostomy. Which action by the nurse is most appropriate? Emptying the pouch if it is more than one-third to one-half full The nurse will irrigate a patient’s nasogastric (NG) tube. Which action should the nurse take? Immediately aspirate after instilling fluid. The nurse administers a cathartic to a patient. Which finding helps the nurse determine that the cathartic has a therapeutic effect? Has a bowel movement. An older adult’s perineal skin is dry and thin with mild excoriation. When providing hygiene care after episodes of diarrhea, what should the nurse do? Apply a skin protective ointment after perineal care. Which action will the nurse take to reduce the risk of excoriation to the mucosal lining of the patient’s nose from a nasogastric tube? Lubricate the nares with water-soluble lubricant. A nurse is providing discharge teaching for a patient who is going home with a guaiac test. Which statement by the patient indicates the need for further education? “If I get a blue color that means the test is negative.” A nurse is preparing to lavage a patient in the emergency department for an overdose. Which tube should the nurse obtain? Ewald The nurse is caring for a patient with Clostridium difficile. Which nursing actions will have the greatest impact in preventing the spread of the bacteria? Proper hand hygiene techniques A nurse is performing an assessment on a patient who has not had a bowel movement in 3 days. The nurse will expect which other assessment finding? Hypoactive bowel sounds A nurse is caring for a patient who has had diarrhea for the past week. Which additional assessment finding will the nurse expect? Decreased skin turgor The nurse is caring for a patient who had a colostomy placed yesterday. The nurse should report which assessment finding immediately? Stoma is purple. A patient is receiving a neomycin solution enema. Which primary goal is the nurse trying to achieve? Prevent colon infection A guaiac test is ordered for a patient. Which type of blood is the nurse checking for in this patient’s stool? Microscopic A patient is receiving opioids for pain. Which bowel assessment is a priority? Constipation Which nutritional instruction is a priority for the nurse to advise a patient about with an ileostomy? Increase fluid intake. A nurse is preparing a bowel training program for a patient. Which actions will the nurse take? (Select all that apply.) Record times when the patient is incontinent. Help the patient to the toilet at the designated time. Maintain normal exercise within the patient’s physical ability. Choose a time based on the patient’s pattern to initiate defecation-control measures. A nurse is teaching a health class about colorectal cancer. Which information should the nurse include in the teaching session? (Select all that apply.) A risk factor is smoking. A risk factor is high intake of animal fats or red meat. A warning sign is rectal bleeding. A warning sign is a sense of incomplete evacuation. Chapter 48—Skin Integrity & Wound Care The nurse is working on a medical-surgical unit that has been participating in a research project associated with pressure ulcers. Which risk factor will the nurse assess for that predisposes a patient to pressure ulcer development? Decreased level of consciousness The nurse is caring for a patient who was involved in an automobile accident 2 weeks ago. The patient sustained a head injury and is unconscious. Which priority element will the nurse consider when planning care to decrease the development of a decubitus ulcer? Pressure Which nursing observation will indicate the patient is at risk for pressure ulcer formation? The patient has fecal incontinence. The wound care nurse visits a patient in the long-term care unit. The nurse is monitoring a patient with a Stage III pressure ulcer. The wound seems to be healing, and healthy tissue is observed. How should the nurse document this ulcer in the patient’s medical record? Healing Stage III pressure ulcer The nurse is admitting an older patient from a nursing home. During the assessment, the nurse notes a shallow open reddish, pink ulcer without slough on the right heel of the patient. How will the nurse stage this pressure ulcer? Stage II The nurse is completing a skin assessment on a patient with darkly pigmented skin. Which item should the nurse use first to assist in staging an ulcer on this patient? Halogen light The nurse is caring for a patient with a Stage IV pressure ulcer. Which type of healing will the nurse consider when planning care for this patient? Full-thickness wound repair The nurse is caring for a group of patients. Which patient will the nurse see first? A patient with appendicitis using a heating pad The nurse is caring for a patient who is experiencing a full-thickness repair. Which type of tissue will the nurse expect to observe when the wound is healing? Granulation The nurse is caring for a patient who has experienced a laparoscopic appendectomy. For which type of healing will the nurse focus the care plan? Primary intention The nurse is caring for a patient in the burn unit. Which type of wound healing will the nurse consider when planning care for this patient? Secondary intention A nurse is assessing a patient’s wound. Which nursing observation will indicate the wound healed by secondary intention? Scarring that may be severe The nurse is caring for a patient who has experienced a total abdominal hysterectomy. Which nursing observation of the incision will indicate the patient is experiencing a complication of wound healing? The site has a mass, bluish in color. A nurse is caring for a postoperative patient. Which finding will alert the nurse to a potential wound dehiscence? Report by patient that something has given way A patient has developed a pressure ulcer. Which laboratory data will be important for the nurse to check? Albumin A nurse is caring for a patient with a wound. Which assessment data will be most important for the nurse to gather with regard to wound healing? Pulse oximetry assessment The nurse is caring for a patient with a healing Stage III pressure ulcer. Upon entering the room, the nurse notices an odor and observes a purulent discharge, along with increased redness at the wound site. What is the next best step for the nurse? Complete the head-to-toe assessment, including current treatment, vital signs, and laboratory results. The nurse is collaborating with the dietitian about a patient with a Stage III pressure ulcer. Which nutrient will the nurse most likely increase after collaboration with the dietitian? Protein The nurse is completing an assessment on a patient who has a Stage IV pressure ulcer. The wound is odorous, and a drain is currently in place. Which statement by the patient indicates issues with self-concept? “I am ready for my bath and linen change right now since this is awful.” A patient presents to the emergency department with a laceration of the right forearm caused by a fall. After determining that the patient is stable, what is the next best step for the nurse to take? Inspect the wound for bleeding. The nurse is caring for a patient on the medical-surgical unit with a wound that has a drain and a dressing that needs changing. Which action should the nurse take first? Provide analgesic medications as ordered. The nurse is caring for a patient who has a wound drain with a collection device. The nurse notices that the collection device has a sudden decrease in drainage. Which action will the nurse take next? Call the health care provider; a blockage is present in the tubing. The nurse is caring for a patient who has a Stage IV pressure ulcer with grafted surgical sites. Which specialty bed will the nurse use for this patient? Air-fluidized The nurse is caring for a patient with a pressure ulcer on the left hip. The ulcer is black. Which next step will the nurse anticipate? Debride the wound. The nurse is caring for a patient with a healing Stage III pressure ulcer. The wound is clean and granulating. Which health care provider’s order will the nurse question? Irrigate with Dakin’s solution. The nurse is completing an assessment of the patient’s skin’s integrity. Which assessment is the priority? Pressure points The nurse is completing a skin risk assessment using the Braden Scale. The patient has slight sensory impairment, has skin that is rarely moist, walks occasionally, and has slightly limited mobility, along with excellent intake of meals and no apparent problem with friction and shear. Which score will the nurse document for this patient? 20 The nurse is caring for a surgical patient. Which intervention is most important for the nurse to complete to decrease the risk of pressure ulcers and encourage the patient’s willingness and ability to increase mobility? Provide analgesic medication as ordered. The nurse is caring for a patient with a Stage IV pressure ulcer. Which nursing diagnosis does the nurse add to the care plan? Impaired skin integrity The nurse collects the following assessment data: right heel with reddened area that does not blanch. Which nursing diagnosis will the nurse assign to this patient? Ineffective peripheral tissue perfusion The nurse is caring for a patient who is immobile. The nurse wants to decrease the formation of pressure ulcers. Which action will the nurse take first? Determine the patient’s risk factors. The medical-surgical acute care patient has received a nursing diagnosis of Impaired skin integrity. Which health care team member will the nurse consult? Registered dietitian The nurse is caring for a patient with a Stage II pressure ulcer and has assigned a nursing diagnosis of Risk for infection. The patient is unconscious and bedridden. The nurse is completing the plan of care and is writing goals for the patient. Which is the best goal for this patient? The patient will remain free of odorous or purulent drainage from the wound. The nurse is caring for a group of patients. Which task can the nurse delegate to the nursing assistive personnel? Applying an elastic bandage to a medical-surgical patient The nurse is performing a moist-to-dry dressing. The nurse has prepared the supplies, solution, and removed the old dressing. In which order will the nurse implement the steps, starting with the first one? Apply sterile gloves > Apply > Assess > Moisten > Gently > Loosely > Cover // AAsMGLC Assess wound and surrounding skin > Moisten gauze with prescribed solution > Gently wring out excess solution and unfold > Loosely pack until all wound surfaces are in contact with gauze > Cover and secure topper dressing The nurse is caring for a patient who has suffered a stroke and has residual mobility problems. The patient is at risk for skin impairment. Which initial actions should the nurse take to decrease this risk? Use gentle cleansers, and thoroughly dry the skin. The nurse is caring for a patient who is at risk for skin impairment. The patient is able to sit up in a chair. The nurse includes this intervention in the plan of care. How long should the nurse schedule the patient to sit in the chair? Less than 2 hours The nurse is caring for a patient who is immobile and is at risk for skin impairment. The plan of care includes turning the patient. Which is the best method for repositioning the patient? Utilize a transfer device to lift the patient. A nurse is assigned most of the patients with pressure ulcers. The nurse leaves the pressure ulcer open to air and does not apply a dressing. To which patient did the nurse provide care? A patient with a clean Stage I The nurse is caring for a patient with a wound. The patient appears anxious as the nurse is preparing to change the dressing. Which action should the nurse take? Explain the procedure. The nurse is cleansing a wound site. As the nurse administers the procedure, which intervention should be included? Cleanse in a direction from the least contaminated area. The nurse is caring for a patient after an open abdominal aortic aneurysm repair. The nurse requests an abdominal binder and carefully applies the binder. Which is the best explanation for the nurse to use when teaching the patient the reason for the binder? It supports the abdomen. The nurse is caring for a postoperative medial meniscus repair of the right knee. Which action should the nurse take to assist with pain management? Apply ice. The patient has a risk for skin impairment and has a 15 on the Braden Scale upon admission. The nurse has implemented interventions. Upon reassessment, which Braden score will be the best sign that the risk for skin breakdown is removed? 23 The nurse is caring for a patient with a surgical incision that eviscerates. Which actions will the nurse take? (Select all that apply.) Place moist sterile gauze over the site. Contact the surgical team. Monitor for shock. The nurse is caring for a patient with a wound healing by full-thickness repair. Which phases will the nurse monitor for in this patient? (Select all that apply.) Hemostasis Maturation Inflammatory Inflammatory The nurse is completing a skin assessment on a medical-surgical patient. Which nursing assessment questions should be included in a skin integrity assessment? (Select all that apply.) “Can you easily change your position?” “Do you have sensitivity to heat or cold?” “How often do you need to use the toilet?” “Is movement painful?” The nurse is caring for a patient with potential skin breakdown. Which components will the nurse include in the skin assessment? (Select all that apply.) Hyperemia Induration Blanching Temperature of skin The nurse is caring for a patient who will have both a large abdominal bandage and an abdominal binder. Which actions will the nurse take before applying the bandage and binder? (Select all that apply.) Cover exposed wounds. Assess the condition of current dressings. Inspect the skin for abrasions and edema. Assess the skin at underlying areas for circulatory impairment. The nurse is updating the plan of care for a patient with impaired skin integrity. Which findings indicate achievement of goals and outcomes? (Select all that apply.) Skin is intact with no redness or swelling. Nonblanchable erythema is absent. No injuries to the skin and tissues are evident. Granulation tissue is present. The nurse is caring for patients who need wound dressings. Match the type of dressing the nurse applies to its description. Absorbs drainage through the use of exudate absorbers in the dressing Hydrocolloid Very soothing to the patient and do not adhere to the wound bed Hydrogel Barrier to external fluids/bacteria but allows wound to “breathe” Transparent Manufactured from seaweed and comes in sheet and rope form Calcium alginate Oldest and most common absorbent dressing Guaze Chapter 49 – Sensory Alterations A nurse is administering a vaccine to a child who is visually impaired. After the needle enters the arm, the child says, “Ow, that was sharp!” How will the nurse interpret the finding when the child said that it was sharp? The child’s perception is intact. A nurse is describing the transmission of sound to a patient. In which order will the nurse list the pathway of sound, beginning with the first structure? Eardrum > Bony ossicles > Oval window > Perilymph > Eighth cranial nerve // EBOP8 A nurse is caring for a patient with presbycusis. Which assessment finding indicates an adaptation to the sensory deficit? The patient turns one ear toward the nurse during conversation. The nurse will be most concerned about the risk of malnutrition for a patient with which sensory deficit? Xerostomia A nurse is caring for an older adult. Which sensory change will the nurse identify as normal during the assessment? Impaired night vision A nurse is caring for an older-adult patient who was in a motor vehicle accident because the patient thought the stoplight was green. The patient asks the nurse “Should Istop driving?” Which response by the nurse is most therapeutic? “No, as you age, you lose the ability to see colors. You need to think about stoplights in a new way. If the top is lit up, it means stop, and if the bottom is lit up, it means go.” A nurse is caring for a patient who recently had a stroke and is going to be discharged at the end of the week. The nurse notices that the patient is having difficulty with communication and becomes tearful at times. Which intervention will the nurse include in the patient’s plan of care? Teach the patient about special assistive devices. A patient has both hearing and visual sensory impairments. Which psychological nursing diagnosis will the nurse add to the care plan? Social isolation During an assessment of a patient, the nurse finds the patient experiences vertigo. Which sensory deficit will the nurse assess further? Balance deficit A home health nurse is assembling a puzzle with an older-adult patient and notices that the patient is having difficulty connecting two puzzle pieces. Which aspect of sensory deprivation will the nurse document as being most affected? Perceptual Which assessment question should the nurse ask to best understand how visual alterations are affecting the patient’s self-care ability? “Are you able to prepare a meal or write a check?” A nurse is assessing cognitive functioning of a patient. Which action will the nurse take? Administer a Mini-Mental State Examination (MMSE). The nurse is using the Snellen chart. Which patient is the nurse assessing? A patient who frequently reports the incorrect time from the clock across the room. A new nurse is caring for a patient who is undergoing chemotherapy for cancer. The patient is becoming malnourished because nothing tastes good. Which recommendation by the nurse will be most appropriate for this patient? “Rinse your mouth several times a day to hydrate your taste buds.” The nurse is creating a plan of care for a patient with glaucoma. Which nursing diagnosis will the nurse include in the care plan to address a safety complication of the sensory deficit? Risk for falls The nurse is caring for a patient who is having difficulty understanding the written and spoken word. Which type of aphasia will the nurse report to the oncoming shift? Receptive The nurse is caring for a patient with conductive hearing loss resulting from prolonged cerumen impaction. Which intervention by the nurse is most important in establishing effective communication with the patient? Speaking with hands, face, and expressions The home health nurse is caring for a patient with tactile and visual deficits. The nurse is concerned about injury related to inability to feel harmful stimuli and teaches the patient safety strategies to maintain independence. Which action by the patient indicates successful learning? Places colored stickers on faucet handles to indicate temperature. A nurse is working to prevent blindness. Which preventive action is a priority? Include rubella and syphilis screening in the preconception care plan. The nurse is caring for a patient in acute respiratory distress. The patient has multiple monitoring systems on that constantly beep and make noise. The patient is becoming agitated and frustrated over the inability to sleep. Which action by the nurse is most appropriate for this patient? Provide the patient with earplugs. The nurse is caring for a patient with expressive aphasia from a traumatic brain injury. Which goal will the nurse include in the plan of care? Patient will communicate nonverbally. The nurse is caring for a group of patients and is monitoring for sensory deprivation. Which patient will the nurse monitor most closely? A patient on the unit with tuberculosis on airborne precautions A nurse is caring for an older-adult patient on bed rest with potential sensory deprivation. Which action will the nurse take? Offer the patient a back rub. The nurse is caring for a patient who is a well-known surgeon at the hospital. The nurse notices the patient becoming more agitated and withdrawn with each group of surgeon visitors. The nurse and patient agree to place a “Do not disturb” sign on the door. A few hours later, the nurse notices a surgeon who is not involved in the patient’s care attempting to enter the room. Which response by the nurse is most appropriate? Firmly explain that the patient does not wish to have visitors at this time. The nurse is caring for a patient who is recovering from a traumatic brain injury and frequently becomes disoriented to everything except location. Which nursing intervention will the nurse add to the care plan to reduce confusion? Keep a day-by-day calendar at the patient’s bedside. A nurse is establishing a relationship with the patient who is severely visually impaired and is teaching the patient how to contact the nurse for assistance. Which action will the nurse take? Place a raised Braille sticker on the call button. The nurse is caring for a patient who is taking gentamicin for an infection. Which assessment is a priority? Hearing A nurse is teaching a patient about vision. In which order will the nurse describe the pathway for vision, beginning with the first structure? Cornea > Pupil > Lens > Retina > Optic nerve //// CPLRO A nurse is caring for a patient with a right hemisphere stroke and partial paralysis. Which action by the nursing assistive personnel (NAP) will cause the nurse to praise the NAP? Dressing the left side first A home care nurse is inspecting a patient’s house for safety issues. Which findings will cause the nurse to address the safety problems? (Select all that apply.) Stairway faintly lit Scatter rugs in the kitchen Absence of smoke alarms NCLEX REVIEW QUESTIONS What would you expect to hear from severe left-sided heart failure? Crackles Which of the following valves must blood go through to go from right atrium into right ventricle? Tricuspid valve A nurse is caring for 3 different patients. One has TB, the other has varicella, third has rubella which patient will the nurse incorporate droplet precautions for? Patient 3 (Rubella) Which of the following sleep disorders poses the greatest risk for patients? Obstructive sleep apnea Which of these are appropriate nursing intervention to prevent sensory deprivation (Select all that apply) Provide coloring books Reading to patient Massaging patient An 84-year-old male is admitted to a long-term facility for a generalized weakness. He has peripheral edema and stage II pressure injury (on his left foot/ heel). In order to check for the skin turgor, where would be the best site for an assessment? Sternum Match description to pressure injury stages Non blanchable discoloration with skin intact Stage I Full thickness loss with exposed bone, tissue and cartilage with slough and eschar Stage IV Partial thickness loss with a serum- filled blister Stage II Excessive eschar and slough w/ obscure wound bed Unstageable Full thickness loss with exposed adipose tissue with some slough and eschar Stage III Which client would have the first priority to be seen? A 5-year-old bed-bound client with a stage I pressure injury What is the best method to prevent medication errors? Rights of administration What organ is the primary side of drug metabolism? Liver Which electrolyte aids in the metabolism of carbohydrates and protein. It also acts on the CVS producing vasodilation. Mg2+ After drawing lab values for a patient admitted into the ER, the nurse evaluated the following ABG’s PH=7.31 HCO3= 18, PaCO2= 36. What could the patient be experiencing? Metabolic acidosis While interviewing a patient you notice they are disheveled and withdrawn. The patient states they recently failed a class and having a hard time. Which variable of self concept best reflects the patients response? (Select all that apply) Self-Esteem Self-actualization [Show More]

Last updated: 1 year ago

Preview 1 out of 107 pages

Add to cart

Instant download

document-preview

Buy this document to get the full access instantly

Instant Download Access after purchase

Add to cart

Instant download

Reviews( 0 )

$12.50

Add to cart

Instant download

Can't find what you want? Try our AI powered Search

OR

REQUEST DOCUMENT
37
0

Document information


Connected school, study & course


About the document


Uploaded On

Jul 22, 2021

Number of pages

107

Written in

Seller


seller-icon
masterguide

Member since 3 years

0 Documents Sold


Additional information

This document has been written for:

Uploaded

Jul 22, 2021

Downloads

 0

Views

 37

Document Keyword Tags

Recommended For You

Get more on EXAM »

$12.50
What is Browsegrades

In Browsegrades, a student can earn by offering help to other student. Students can help other students with materials by upploading their notes and earn money.

We are here to help

We're available through e-mail, Twitter, Facebook, and live chat.
 FAQ
 Questions? Leave a message!

Follow us on
 Twitter

Copyright © Browsegrades · High quality services·